Welcome to
Science a GoGo's
Discussion Forums
Please keep your postings on-topic or they will be moved to a galaxy far, far away.
Your use of this forum indicates your agreement to our terms of use.
So that we remain spam-free, please note that all posts by new users are moderated.


The Forums
General Science Talk        Not-Quite-Science        Climate Change Discussion        Physics Forum        Science Fiction

Who's Online Now
0 members (), 388 guests, and 4 robots.
Key: Admin, Global Mod, Mod
Latest Posts
Top Posters(30 Days)
Previous Thread
Next Thread
Print Thread
Page 1 of 9 1 2 3 4 5 6 7 8 9
#55406 02/11/16 12:42 PM
Joined: Aug 2010
Posts: 3,570
B
Bill S. Offline OP
Megastar
OP Offline
Megastar
B
Joined: Aug 2010
Posts: 3,570
What is a second? That’s an easy one; a second is defined as 9 192 631 770 oscillations of a caesium 133 atom.

That’s all settled, then; or is it?

Does a second have any independent existence, or is it just something that has emerged from arbitrary divisions of years and days?

Compare, for example, a second and a proton. It seems quite reasonable to assume that a proton has an existence that is independent of our measurements. It is probably safe to assume that the diameter of a proton would be about 1.7536 femtometres, whether or not we ever measured, or even thought about it.

Can we say the same for a second? Would a second exist as anything in its own right if it had never been defined? If not, does that tell us anything about the “reality” of time? One thing we know about the second is that it is the SI base unit for the measurement of time. So, for example, if the base unit of time has no independent existence, can time be said to have independent existence?


There never was nothing.
.
Bill S. #55407 02/11/16 04:22 PM
Joined: May 2011
Posts: 2,819
O
Megastar
Offline
Megastar
O
Joined: May 2011
Posts: 2,819
You aren't being very consistent your proton which you are happy to say exists can only be defined by a wave. I assume you aren't going to try and cling to the classical rubbish. So if your proton can only be described by a wave then time must exist for the wave to exist (wave = movement in time BY DEFINITION).

Same logic as you applied to the proton it doesn't matter whether you measured it or not, it has to exist to the same level the proton does.

So either both exist or neither exist and that is reality .... QED.

Given release of direct detection of gravity waves today we can even say that anywhere gravity exists then time must exist ... GR demands it !!

So this idea is now up against GR and QM which both say no smile

For science time exists from the moment of the big bang, it isn't up for conjecture to laymen. Personally I find the whole idea of playing with the concept naive and foolish. A related question would be does the universe exist if Bill S wasn't born, how would he know.

Last edited by Orac; 02/11/16 05:51 PM.

I believe in "Evil, Bad, Ungodly fantasy science and maths", so I am undoubtedly wrong to you.
Bill S. #55412 02/12/16 12:12 AM
Joined: Mar 2006
Posts: 4,136
P
Megastar
Offline
Megastar
P
Joined: Mar 2006
Posts: 4,136
Quote:
Would a second exist as anything in its own right if it had never been defined?


would a kilogram exist as anything in its own right if it had never been defined?

would a meter exist as anything in its own right if it had never been defined?

would speed exist as anything in its own right if it had never been defined?

would temperature exist as anything in its own right if it had never been defined?

measurements are simply measurements and that is all that
they are , you cannot manipulate a measurement because it has no physical properties.

so my logical layman answer is NO.



3/4 inch of dust build up on the moon in 4.527 billion years,LOL and QM is fantasy science.
Orac #55413 02/12/16 12:18 AM
Joined: Mar 2006
Posts: 4,136
P
Megastar
Offline
Megastar
P
Joined: Mar 2006
Posts: 4,136
Quote:
So either both exist or neither exist and that is reality


that's pure uncredible BS orac.

photons will continue to exist even after all men have
gone and taken their measurements with them.

your stupid brand of science is becoming more than
just plain stupid its bordering on idiocy.


3/4 inch of dust build up on the moon in 4.527 billion years,LOL and QM is fantasy science.
Orac #55414 02/12/16 12:32 AM
Joined: Mar 2006
Posts: 4,136
P
Megastar
Offline
Megastar
P
Joined: Mar 2006
Posts: 4,136
Quote:
Given release of direct detection of gravity waves today we can even say that anywhere gravity exists then time must exist ... GR demands it !!


time does not exist anywhere , theres simply no such THING
as time , time is only a measurement.

GR is most likely and most uncredibly demanding more illusions to support some previous illusions , so I and we can now and always have said that GR itself is an illusion.


3/4 inch of dust build up on the moon in 4.527 billion years,LOL and QM is fantasy science.
Bill S. #55427 02/12/16 10:41 PM
Joined: Aug 2010
Posts: 3,570
B
Bill S. Offline OP
Megastar
OP Offline
Megastar
B
Joined: Aug 2010
Posts: 3,570
Quote:
You aren't being very consistent your proton which you are happy to say exists can only be defined by a wave.


I am never happy to say that anything exists. If I make such a statement you can generally assume that it is “shorthand” for something like “We detect what we interpret as its physical existence”.

Quote:
So if your proton can only be described by a wave then time must exist for the wave to exist


As you say: it can only be described by a wave. This does not guarantee that it is a wave. My understanding is that quantum objects are neither particles nor waves.

Quote:
(wave = movement in time BY DEFINITION).


Even disregarding any doubt that the proton might actually be a wave; all this establishes is that change takes place; time might only be the tool we use to make sense of what we call change.


There never was nothing.
Bill S. #55447 02/15/16 02:46 AM
Joined: May 2011
Posts: 2,819
O
Megastar
Offline
Megastar
O
Joined: May 2011
Posts: 2,819
Originally Posted By: Bill S.
all this establishes is that change takes place

Think very very very very very very very carefully about what you just said.

Hint: try to define time

Is it possible for anything to change without time? smile

Response:
We have a state (A) of an item, and it evolves to a new state (B). For science the evolution of A=>B is called time. If no time exists you will never observe the evolution. Does that help with your statement above?

A state can be captured easily in an image, a time evolution takes a minimum of two images (AKA measurements) ... lets show you

State A:


State B:

Last edited by Orac; 02/15/16 06:54 AM.

I believe in "Evil, Bad, Ungodly fantasy science and maths", so I am undoubtedly wrong to you.
Orac #55455 02/15/16 01:41 PM
Joined: Mar 2006
Posts: 4,136
P
Megastar
Offline
Megastar
P
Joined: Mar 2006
Posts: 4,136
Quote:
try to define time


I will as soon as you define.

temperature
speed
distance

does temperature exist as anything other than a measurement ?
does speed exist as anything other than a measurement ?
does distance exist as anything other than a measurement?

now define time as anything other than a measurement.

Quote:
Is it possible for anything to change without time?


time does not exist.

you should have said

Is it possible for anything to change without the passage
of a measurement of time?

you and your version of science wants time to exist as
more than just a measurement in order to pacify some
previous or future illusion , that cant happen except inside of an illusion.



3/4 inch of dust build up on the moon in 4.527 billion years,LOL and QM is fantasy science.
paul #55461 02/15/16 06:02 PM
Joined: May 2011
Posts: 2,819
O
Megastar
Offline
Megastar
O
Joined: May 2011
Posts: 2,819
Originally Posted By: paul

does temperature exist as anything other than a measurement ?
does speed exist as anything other than a measurement ?
does distance exist as anything other than a measurement?

now define time as anything other than a measurement.

So time is the same as temperature, speed and distance just a measurement?
Do that and you will score an own goal, again Paulsmile

Originally Posted By: paul
time does not exist.

Well if time doesn't exist then nor does temperature, speed and distance if they are just measurements ... see your comment above.

You found the hole, I was trying to make Bill S see smile

Originally Posted By: paul
you and your version of science wants time to exist as more than just a measurement in order to pacify some
previous or future illusion , that cant happen except inside of an illusion

No I am really happy to have time just as a measurement. See you need concentrate on the argument not just disagree with me.

However I think you better go back and make temperature, speed and distance something more than just a measurement to save your classical world smile

Lets show why you need to save it:
Paul, can I ask does the Moon exist if you aren't measuring it? laugh

Last edited by Orac; 02/15/16 06:20 PM.

I believe in "Evil, Bad, Ungodly fantasy science and maths", so I am undoubtedly wrong to you.
paul #55462 02/15/16 06:38 PM
Joined: Aug 2010
Posts: 3,570
B
Bill S. Offline OP
Megastar
OP Offline
Megastar
B
Joined: Aug 2010
Posts: 3,570
Quote:
Is it possible for anything to change without the passage
of a measurement of time?


I'm maintaining my position on the fence, here, so I can take a shot at either side, for the purpose of learning, of course.:)

"... a measurement of time" implies that time is something that can be measured. Is that what you meant to say?


There never was nothing.
Orac #55463 02/15/16 06:51 PM
Joined: Aug 2010
Posts: 3,570
B
Bill S. Offline OP
Megastar
OP Offline
Megastar
B
Joined: Aug 2010
Posts: 3,570
Quote:
We have a state (A) of an item, and it evolves to a new state (B). For science the evolution of A=>B is called time. If no time exists you will never observe the evolution


Surely the evolution A -> B is called change; time is what we use to make sense of our observation of change.

In our 3+1D Universe change is essential, therefore we have to have time in order to give it meaning. Does that mean that time exists outside the realm of change? Thought is a "real" thing, but would it exist without a thinker?

BTW, Orac, that's not you in the top pic, is it? laugh

Last edited by Bill S.; 02/15/16 06:52 PM.

There never was nothing.
Orac #55469 02/15/16 11:43 PM
Joined: Mar 2006
Posts: 4,136
P
Megastar
Offline
Megastar
P
Joined: Mar 2006
Posts: 4,136
Quote:
Well if time doesn't exist then nor does temperature, speed and distance if they are just measurements ... see your comment above.


time , temperature , speed and distance do not exist.

they never have existed , they never will exist.

they are only measurements.


3/4 inch of dust build up on the moon in 4.527 billion years,LOL and QM is fantasy science.
Bill S. #55470 02/15/16 11:57 PM
Joined: Mar 2006
Posts: 4,136
P
Megastar
Offline
Megastar
P
Joined: Mar 2006
Posts: 4,136
Quote:
Is it possible for anything to change without the passage
of a measurement of time?


suppose you wanted to measure the thickness of a wooden
board with a measuring stick , you could not even reach
out to pick up the measuring stick unless a measurement
of time passes.

the wooden board exist , the measurement stick exist
you exist , but if you could measure the thickness of
the wooden board the measurement would not exist.


Quote:
"... a measurement of time" implies that time is something that can be measured. Is that what you meant to say?


https://en.wikipedia.org/wiki/Measurement

Quote:
Measurement is the assignment of a number to a characteristic of an object or event, which can be compared with other objects or events


for instance a 1 ounce gold bar has a measurement of
1 ounce troy weight.

the gold exist , the 1 oz is simply a measurement of the
amount of gold in the gold bar , the measurement does not exist.

Quote:
Is it possible for anything to change without the passage
of a measurement of time?


what Im saying is that nothing can move without
the passage of time and time cannot be manipulated
it does not exist it has no physical properties that
can be adjusted.

it just passes.
it never slows down.
it never speeds up.
it never stops.

it cant because it does not exist.




3/4 inch of dust build up on the moon in 4.527 billion years,LOL and QM is fantasy science.
Bill S. #55475 02/16/16 02:02 AM
Joined: May 2011
Posts: 2,819
O
Megastar
Offline
Megastar
O
Joined: May 2011
Posts: 2,819
Originally Posted By: Bill S.
"... a measurement of time" implies that time is something that can be measured. Is that what you meant to say?

Bingo and Paul took that stance by calling it a measurement.

The problem for him is you have to give other properties to his other quantities so they persist when you aren't measuring .. so they become more than just a measurement. I see he has responded but I haven't read if he has worked that out yet.


I believe in "Evil, Bad, Ungodly fantasy science and maths", so I am undoubtedly wrong to you.
Bill S. #55476 02/16/16 02:11 AM
Joined: May 2011
Posts: 2,819
O
Megastar
Offline
Megastar
O
Joined: May 2011
Posts: 2,819
Originally Posted By: Bill S.
Surely the evolution A -> B is called change; time is what we use to make sense of our observation of change.

Agree

Quote:
In our 3+1D Universe change is essential, therefore we have to have time in order to give it meaning.

Woot and there we have it ... your universe is defined by change

Quote:
Does that mean that time exists outside the realm of change?

Now you got the next part of the extension, there is no way to tackle that is there because of the way we defined time.

Quote:
Thought is a "real" thing, but would it exist without a thinker?

Now you have indeed worked the connection, so the question you are really asking is does time exist without our universe which is defined by change.

To take time outside the universe I need to redefine it in a non our universe way. So it is like trying to define thinking without relating it to a thinker. Want to have a crack at doing it?

Originally Posted By: Bill S.
BTW, Orac, that's not you in the top pic, is it? laugh

LOL so mean.

Last edited by Orac; 02/16/16 02:13 AM.

I believe in "Evil, Bad, Ungodly fantasy science and maths", so I am undoubtedly wrong to you.
paul #55477 02/16/16 02:15 AM
Joined: May 2011
Posts: 2,819
O
Megastar
Offline
Megastar
O
Joined: May 2011
Posts: 2,819
Originally Posted By: paul
time , temperature , speed and distance do not exist.

they never have existed , they never will exist.

they are only measurements.

Paul you said you don't believe in QM smile

So the moon doesn't exist except when you look at it !!

Welcome to the darkside my new little QM convert. I can't believe you walked into that head first and yes I am the devil. However to be fair to me, you joined the conversation and walked yourself into a hole I didn't do anything.

I bet you want to revise some of your answers now my little QM believer laugh

I really am not a mean person so lets give you a hint, the usual way out in classical physics is to add some sort of persistence to measurement so it exists as some "property". So your quantities velocity, temp etc need to exist as more than just a measurement. So classical physics does that to all the properties except time. Why not time, well I will leave you to think about that.

Last edited by Orac; 02/16/16 02:54 AM.

I believe in "Evil, Bad, Ungodly fantasy science and maths", so I am undoubtedly wrong to you.
paul #55478 02/16/16 03:01 AM
Joined: Aug 2010
Posts: 3,570
B
Bill S. Offline OP
Megastar
OP Offline
Megastar
B
Joined: Aug 2010
Posts: 3,570
Quote:
it just passes......

....it does not exist.


How can something that doesn't exist do any passing, or anything else?


There never was nothing.
Orac #55479 02/16/16 03:04 AM
Joined: Aug 2010
Posts: 3,570
B
Bill S. Offline OP
Megastar
OP Offline
Megastar
B
Joined: Aug 2010
Posts: 3,570
Quote:
Woot and there we have it ... your universe is defined by change


That I have never denied.


There never was nothing.
Bill S. #55480 02/16/16 03:14 AM
Joined: May 2011
Posts: 2,819
O
Megastar
Offline
Megastar
O
Joined: May 2011
Posts: 2,819
Originally Posted By: Bill S.
How can something that doesn't exist do any passing, or anything else?

Bad boy I wanted Paul to work that out ... I knew you would get it.

The downside of framing two answers in one post frown

Last edited by Orac; 02/16/16 03:16 AM.

I believe in "Evil, Bad, Ungodly fantasy science and maths", so I am undoubtedly wrong to you.
Bill S. #55484 02/16/16 05:50 AM
Joined: Aug 2007
Posts: 84
P
Member
Offline
Member
P
Joined: Aug 2007
Posts: 84
"Does a second have any independent existence, or is it just something that has emerged from arbitrary divisions of years and days?"


A second is a measurement of time, a unit defined by us.

Time (perhaps a tool we use to make sense of what we term change) exists. Unless you’re a
photon moving at c, as I understand they experience no passing of time.

Since we exist in space-time, the faster we move in space the slower we move in time (time dilation).

Time is measured in its own frame of reference, so if you have two observers in different FOR they
will not measure a time interval the same.

Photons travel at c in all FOR no matter how fast we are moving.

pokey #55488 02/16/16 06:21 AM
Joined: May 2011
Posts: 2,819
O
Megastar
Offline
Megastar
O
Joined: May 2011
Posts: 2,819
You are sort of correct pokey and caught up with the problem we have everyone trying to answer

The one most people will struggle with and may argue with you is this one
Originally Posted By: pokey
Time (perhaps a tool we use to make sense of what we term change) exists. Unless you’re a photon moving at c, as I understand they experience no passing of time.

Paul went down the path of making time not existing. He found the easy solution to change everything else into measurements and not existing, which he promptly did. In QM that idea is a form of holographic principle, that things only exist based on measurement and hence my little joke with Paul who is now a convert. There is a very famous exchange with Einstein over the existence of the moon.

So you are going down the time exists path. So your question I would like you to front is a little different. If time exists how can it be stopped for the photon (that is appear not to exist) and then the extension of that is can it go backwards? If it can't go backwards what stops it?

Extension thinking excercise:
The interesting part to ponder is time existing or not existing the only two choices. This one usually catches most people out who will swear that it is that black and white. So lets give you a couple of examples in shadows and rainbows. These things generally fall under a category of things called illusions of physical effects. As humans all our senses have illusions that can fool them and we often marvel at them. Can we definitively exclude time from being an illusion of a physical effect?

Last edited by Orac; 02/16/16 06:55 AM.

I believe in "Evil, Bad, Ungodly fantasy science and maths", so I am undoubtedly wrong to you.
pokey #55497 02/16/16 01:25 PM
Joined: Aug 2010
Posts: 3,570
B
Bill S. Offline OP
Megastar
OP Offline
Megastar
B
Joined: Aug 2010
Posts: 3,570
Quote:
...I understand they experience no passing of time.


http://www.askamathematician.com/2011/07/q-does-light-experience-time/#comments

Pokey, I've posted this link before, but in case you missed it, it is worth a look. One interesting aspect is the "Physicist's" reaction, or lack of it.

Last edited by Bill S.; 02/16/16 01:44 PM.

There never was nothing.
Orac #55498 02/16/16 01:43 PM
Joined: Aug 2010
Posts: 3,570
B
Bill S. Offline OP
Megastar
OP Offline
Megastar
B
Joined: Aug 2010
Posts: 3,570
Quote:
The interesting part to ponder is time existing or not existing the only two choices. This one usually catches most people out who will swear that it is that black and white. So lets give you a couple of examples in shadows and rainbows.


If light exists, then shadows and rainbows exist.

The existence, or otherwise, of time is a question that leads to speculation and, possibly, metaphysics. It may not even be a question for science. Science needs time in order to make calculations, so “shut up and calculate” is undoubtedly the most practical approach.

I raised this question because I overheard someone mention a watch that was “accurate to 1 second in 1,000 years”, and I thought: what does that actually mean?

I think there is a possible third option in the existence/non-existence of time debate, but I’m trying to keep off infinity/eternity. We have enough repetitive crackpottery already. smile


There never was nothing.
Bill S. #55499 02/16/16 03:30 PM
Joined: May 2011
Posts: 2,819
O
Megastar
Offline
Megastar
O
Joined: May 2011
Posts: 2,819
That statement you have made is actually very profound but missed the correctness by a tiny whisker. The shadows and rainbow only may exist they would depend on other things as well.

Originally Posted By: Bill S
If light exists, then shadows and rainbows exist.

That is called trinary logic and the form you have given goes like this

C = true, A or B may equal { true or false}
C = false A or B only equal false

your example
light = true, shadow or rainbow may equal { true, false }
light = false shadow or rainbow only ever false

What you will find is you can't change A & B into position C as they have a dependency on C.

Would you like a prompt to think about?

space = true, energy and time may equal {true, false}
space = false, energy and time only ever false.

What trinary logic is really good at is dealing with dependent conditions and help you recognize them. You might want to play around with more quantities in physics.

Yes I also gave you the WHITE ELEPHANT IN THE ROOM you all left out in your discussion of time which was Energy.

I laughed when you said shutup and calculate because you didn't have anything to calculate with in your discussion so far. Not one of you remotely talked about it and for Paul and Pokey it is probably vital because it brings in the persistence.

The motion energy version goes along the lines a body in motion will tend to stay in motion and whatever else Mr Newton said. You have Energy and time connected together with persistence.

Last edited by Orac; 02/16/16 03:56 PM.

I believe in "Evil, Bad, Ungodly fantasy science and maths", so I am undoubtedly wrong to you.
Orac #55500 02/16/16 04:05 PM
Joined: Mar 2006
Posts: 4,136
P
Megastar
Offline
Megastar
P
Joined: Mar 2006
Posts: 4,136
Quote:
Paul you said you don't believe in QM smile

So the moon doesn't exist except when you look at it !!

Welcome to the darkside my new little QM convert. I can't believe you walked into that head first and yes I am the devil. However to be fair to me, you joined the conversation and walked yourself into a hole I didn't do anything.

I bet you want to revise some of your answers now my little QM believer


1) I don't believe in QM , Qm is BS like yourself.

2) the moon exist even if no one were looking at it.

3) I'm waaayyy to realistic to be converted into some
BS illogical realm of fantasy populated by idiots.

Quote:
I really am not a mean person


I never said you were mean , stupid yes , dumber that dirt yes , idiotic (not sure but most likely) yes.





3/4 inch of dust build up on the moon in 4.527 billion years,LOL and QM is fantasy science.
paul #55501 02/16/16 04:07 PM
Joined: May 2011
Posts: 2,819
O
Megastar
Offline
Megastar
O
Joined: May 2011
Posts: 2,819
Settle my little padwan smile

We believe in the same things apparently laugh

You probably need to rethink your answers or are you happy and we call each other bro and hangout together now.

Last edited by Orac; 02/16/16 04:18 PM.

I believe in "Evil, Bad, Ungodly fantasy science and maths", so I am undoubtedly wrong to you.
Bill S. #55502 02/16/16 04:32 PM
Joined: Mar 2006
Posts: 4,136
P
Megastar
Offline
Megastar
P
Joined: Mar 2006
Posts: 4,136
Originally Posted By: paul
it (time) just passes......

....it (time) does not exist.


Originally Posted By: Bill S
How can something that doesn't exist do any passing, or anything else?


(time) used in the context I used is simply four letters
joined together to form a word that is used in communication.

its like (smart) , if (time) exist then (smart) exist.

likewise all other words used to communicate would exist.

(ouch) is also a four letter word used to communicate a feeling of pain or discomfort , if (time) exist then
(ouch) also exist.

every word in every language used for communication
purposes would have to exist if (time) exist.

does (orac) exist independently from the person that
is named (orac) for communication purposes on this forum?

can you think of any other four letter words that do not
have any physical properties that can be said are
attached to them?

what about one letter words such as (a)?

what is a (a)?

(a) is a letter , but it does not exist as anything except
a letter of the alphabet and is used for communication purposes.

(t) is a letter but it does not exist.
(i) is a letter but it does not exist.
(m) is a letter but it does not exist.
(e) is a letter but it does not exist.

time is a word but it has no physical properties and
is only used for communication purposes.

the word (passes) is used for communication purposes also
exactly like the word (time) is used , when using the
two words (time) and (passes) together
as in (time passes) the word (passes) is used as
descriptive text of the word (time).

as in the time seems to be passing so slow when Im at
work these days.

none of the words in the above sentence exist and are
only used for communication purposes.

can you claim that any of the words in the above sentence
have any physical properties attached to them?



3/4 inch of dust build up on the moon in 4.527 billion years,LOL and QM is fantasy science.
Bill S. #55503 02/16/16 04:54 PM
Joined: Aug 2007
Posts: 84
P
Member
Offline
Member
P
Joined: Aug 2007
Posts: 84
Originally Posted By: Bill S.
Quote:
...I understand they experience no passing of time.


http://www.askamathematician.com/2011/07/q-does-light-experience-time/#comments

Pokey, I've posted this link before, but in case you missed it, it is worth a look. One interesting aspect is the "Physicist's" reaction, or lack of it.



Thanks for the link Bill S. Will need to spend some time to sort through the opinions given.

Orac #55504 02/16/16 05:08 PM
Joined: Aug 2007
Posts: 84
P
Member
Offline
Member
P
Joined: Aug 2007
Posts: 84
Can we definitively exclude time from being an illusion of a physical effect? [/quote]

Thanks for the reply, I had to take some sleep time.

I can't definitely exclude anything.

Good grief, Time is going to lead into "double slit" and "delayed choice" isn't it?

Orac #55505 02/16/16 05:13 PM
Joined: Mar 2006
Posts: 4,136
P
Megastar
Offline
Megastar
P
Joined: Mar 2006
Posts: 4,136


Quote:
We believe in the same things apparently


absolutely not ... no way.
you believe that time can dialate , I dont.
you believe that time exist , I don't.
you believe that light has a speed limit, I don't.
you believe that mass has a speed limit, I don't.
you believe so many illogical things that I don't.
you also believe you are intelligent , I don't.


Quote:

You probably need to rethink your answers or are you happy and we call each other bro and hangout together now.


I'll keep my answers , and I'll have to pass on the
hanging out part as well.

so you can stop patting yourself on your own back
as if you had somehow won over a new recruit into your
fantasy realm of illogical nonsense.


3/4 inch of dust build up on the moon in 4.527 billion years,LOL and QM is fantasy science.
paul #55514 02/17/16 03:25 AM
Joined: May 2011
Posts: 2,819
O
Megastar
Offline
Megastar
O
Joined: May 2011
Posts: 2,819
Originally Posted By: paul
you believe that time exist , I don't.

Nope I am happy to have it not exist, so we can get down and hang out like Soul brothers.

Lay some skin on me bro, you are my homeboy now.

Group hug time ... too soon?

Last edited by Orac; 02/17/16 07:36 AM.

I believe in "Evil, Bad, Ungodly fantasy science and maths", so I am undoubtedly wrong to you.
pokey #55515 02/17/16 04:00 AM
Joined: May 2011
Posts: 2,819
O
Megastar
Offline
Megastar
O
Joined: May 2011
Posts: 2,819
Originally Posted By: pokey
I can't definitely exclude anything.

Good grief, Time is going to lead into "double slit" and "delayed choice" isn't it?

Nope we are playing the game of Einstein and GR distilled down to a very layman level.

He connects the trinary logic between space, time and energy and imagines a space where at every point along it time and energy are an illussion of space itself.

For his time every point in his space can be imagined to have a different clock running at a different time dependant only on the properties of space at that point.

For energy it disappears locally at any point in space, you can't measure it at a single point, well it measures zero. You have to pick two points separated in spacetime (a metric) to measure energy between them.

So in Einsteins framework a photon may experience time stopped but that is a local value of zero you are not entitled to project it to any other point in space. That is why time stopping for a photon is not an issue and it doesn't mean time doesn't exist under GR.

Don't get me started on the person falling into a black hole and time stopping that layman always can't get straight in there head smile

What all of you instantly did was take Time as a universe wide global thing (you excluded it being local as in relativity) and none of you connected it to energy.

As you had all gone global and then broken it to exit/non exist, I was trying to work out how the hell I give you all the hint they aren't the only options. Time exist or not is a non issue if it is a local illusion you can take either. Would you argue over a rainbow existing or not because its sort of a nonsense argument.

So I guess we have a way to go in our science education.

Last edited by Orac; 02/17/16 07:23 AM.

I believe in "Evil, Bad, Ungodly fantasy science and maths", so I am undoubtedly wrong to you.
paul #55522 02/17/16 09:49 PM
Joined: Aug 2010
Posts: 3,570
B
Bill S. Offline OP
Megastar
OP Offline
Megastar
B
Joined: Aug 2010
Posts: 3,570
Paul, you should make a good physicist. Most of what you say in #55502 is absolutely right but (in this context) completely useless. It does a few things, but does not address the question of how something that does not exist can be said to “pass”, or do anything else.

What your logic does say is quite interesting.

Quote:
(t) is a letter but it does not exist.
(i) is a letter but it does not exist.
(m) is a letter but it does not exist.
(e) is a letter but it does not exist.


Letters do not exist

Quote:
none of the words in the above sentence exist


Words do not exist, presumably because they are composed of letters that do not exist.

Quote:
time is a word


Therefore it doesn’t exist.

If things designated by words do not exist, what does exist?

Let’s think about that.

That’s it!!! I’m thinking, therefore I exist, but my existence must be wordless.

Everything else is described/defined by words, therefore it does not exist.

Have you hit on the best argument yet for solipsism?


There never was nothing.
Orac #55523 02/18/16 05:21 AM
Joined: Aug 2007
Posts: 84
P
Member
Offline
Member
P
Joined: Aug 2007
Posts: 84
"He connects the trinary logic between space, time and energy and imagines a space where at every point along it time and energy are an illussion of space itself."

So you’re saying that Albert E said:
If Space = true, Energy and Time = false.

Bill S. #55524 02/18/16 02:15 PM
Joined: Mar 2006
Posts: 4,136
P
Megastar
Offline
Megastar
P
Joined: Mar 2006
Posts: 4,136
Quote:
If things designated by words do not exist, what does exist?

Let’s think about that.



things that have physical properties exist...

Quote:

That’s it!!! I’m thinking, therefore I exist, but my existence must be wordless.


so you wouldnt exist if you did not think?
you could certainly exist if you could not think as long as
your bodies life support systems were functioning.

If you think that there is a giant purple hippo jumping up
and down on top of your head would that create the hippo?
try it ... now look up and see if the hippo exist.

basically what you have said is that all words would have never
existed if man did not invent them as a form of communication.

so since the words time , temperature , speed , or any words
that are used only for communication purposes did
not exist until man first existed and invented tools to measure
them with and used them for communication purposes
then time , temperature , speed or any measurement
did not exist before man existed.

drawing the letter A in the sand with a stick does not equate
to the letter A existing , the only thing that would now exist
due to your actions would be the shape of the lines drawn in the sand.

the sand was already there.



3/4 inch of dust build up on the moon in 4.527 billion years,LOL and QM is fantasy science.
paul #55526 02/18/16 10:55 PM
Joined: Aug 2010
Posts: 3,570
B
Bill S. Offline OP
Megastar
OP Offline
Megastar
B
Joined: Aug 2010
Posts: 3,570
Quote:
so you wouldnt exist if you did not think?


Now; there’s something I certainly did not say.

Quote:
If you think that there is a giant purple hippo jumping up
and down on top of your head would that create the hippo?


I very much doubt it; but having had, while driving, had a “Stone Tape” experience of such startling clarity that it caused me to brake very sharply, I tend to have an open mind about reality.

Quote:
basically what you have said is that all words would have never
existed if man did not invent them as a form of communication.


That seems reasonable, but not provable.

Quote:
so since the words time , temperature , speed , or any words
that are used only for communication purposes did
not exist until man first existed and invented tools to measure
them with and used them for communication purposes
then time , temperature , speed or any measurement
did not exist before man existed.


All that says is that the words for these things did not exist before our ancestors invented them. It makes no case for claiming that the things described did not exist before there were words for them.

Quote:
drawing the letter A in the sand with a stick does not equate
to the letter A existing , the only thing that would now exist
due to your actions would be the shape of the lines drawn in the sand.


Remind me; have we defined “existence”?


There never was nothing.
Bill S. #55527 02/19/16 01:09 PM
Joined: Mar 2006
Posts: 4,136
P
Megastar
Offline
Megastar
P
Joined: Mar 2006
Posts: 4,136
Originally Posted By: Bill S

Remind me; have we defined “existence”?


I defined existence when I wrote

Originally Posted By: paul

things that have physical properties exist...


Originally Posted By: Bill S

That seems reasonable, but not provable.


are you saying that the many languages used by humans on
the earth were not invented by the humans on earth?

Quote:

All that says is that the words for these things did not exist before our ancestors invented them. It makes no case for claiming that the things described did not exist before there were words for them.


certainly it does , the things I described do not exist
time, temperature, speed, etc... they have no physical
properties therefore they cannot exist other than in our
own minds (like your stone tape halucination) you certainly
do not believe that the halucination that your mind generated
actually existed anywhere except in your own mind do you?


perhaps we should define the word define before we
define other meanings of other words in order to have
a strong foundation to build upon.

SAGG could use a running open dictionary that we can use
to reference our definitions of words.



3/4 inch of dust build up on the moon in 4.527 billion years,LOL and QM is fantasy science.
paul #55528 02/19/16 01:43 PM
Joined: Mar 2006
Posts: 4,136
P
Megastar
Offline
Megastar
P
Joined: Mar 2006
Posts: 4,136
we can carry this into physics if you like.

motion does not exist
force does not exist
acceleration does not exist
direction does not exist
distance does not exist
length does not exist
depth does not exist
width does not exist
height does not exist
area does not exist
diameter does not exist
radius does not exist
circumference does not exist
spin does not exist
angle does not exist
degree does not exist
radian does not exist
etc ... etc ... etc ...

the words above are words that humans invented to
use to describe objects that do exist and
do have physical properties.


3/4 inch of dust build up on the moon in 4.527 billion years,LOL and QM is fantasy science.
paul #55531 02/20/16 03:43 PM
Joined: Aug 2010
Posts: 3,570
B
Bill S. Offline OP
Megastar
OP Offline
Megastar
B
Joined: Aug 2010
Posts: 3,570
Quote:
things that have physical properties exist...


That might be part of the definition of “things that have physical properties”, but it is not a definition of existence.

Quote:
are you saying that the many languages used by humans on
the earth were not invented by the humans on earth?


From what obscure corner of your imagination did that arise?

I was suggesting that we have no way of knowing if there are other beings that invented language before we did. Even discounting “little green men”; you impress as a person of religious conviction; do you not think God and other spiritual beings might have been ahead of us?

Quote:
certainly it does , the things I described do not exist
time, temperature, speed, etc... they have no physical
properties therefore they cannot exist other than in our
own minds


If you were hit by a car travelling at 5mph, then by the same car travelling at 50mph; would you notice no physical difference; and if you did, would it really be physical or only in your mind?

Quote:
…you certainly
do not believe that the halucination that your mind generated
actually existed anywhere except in your own mind do you?


The experience was totally in my mind, of course. However, I had no way of knowing that some 25 years earlier two, slightly inebriated, young women were walking along that pavement (sidewalk) and that one turned her ankle on the kerb and lurched into the road in front of an oncoming vehicle and was killed. The hallucination was in my mind, but where did the information come from?


There never was nothing.
paul #55532 02/20/16 03:55 PM
Joined: Aug 2010
Posts: 3,570
B
Bill S. Offline OP
Megastar
OP Offline
Megastar
B
Joined: Aug 2010
Posts: 3,570
Let’s take the first example from your list.

Quote:
motion does not exist


Let’s accept that, and move on.

Quote:
the words above are words that humans invented to
use to describe objects that do exist and
do have physical properties.


Would I be right in thinking that this somewhat ambiguous statement is not intended to say that motion is a physical property?


There never was nothing.
Bill S. #55533 02/20/16 04:10 PM
Joined: Aug 2010
Posts: 3,570
B
Bill S. Offline OP
Megastar
OP Offline
Megastar
B
Joined: Aug 2010
Posts: 3,570
Where do I stand on the question of the existence, or non-existence, of time?

Intuitively, I think of time as something we conceived to enable us to make sense of the 3+1D Universe in which we perceive ourselves to exist, and to measure our perception of change in that Universe.

Can time dilate?

Intuitively, I would think not; though our measurement of time may be subject to predictable fluctuations depending on motion or gravity.

At this point, I stop because I have introduced a rational inconsistency. No prizes for spotting it.


There never was nothing.
Bill S. #55536 02/21/16 12:27 AM
Joined: Mar 2006
Posts: 4,136
P
Megastar
Offline
Megastar
P
Joined: Mar 2006
Posts: 4,136
I wont spend alot of time on this reply because it
is taking 5 minutes for any web page to load up.
I can use the tor web browser to view SAGG much quicker
but not to make a reply.

Quote:

If you were hit by a car travelling at 5mph, then by the same car travelling at 50mph; would you notice no physical difference; and if you did, would it really be physical or only in your mind?



there would certainly be a big difference in what I
would notice , but that certainly does not say that
the speed of the car exist.

the car exist but not the speed.
the impact would be greater but the car itself has
not changed.

Quote:

Would I be right in thinking that this somewhat ambiguous statement is not intended to say that motion is a physical property?


yes , motion is not a physical property.

Quote:

Intuitively, I think of time as something we conceived to enable us to make sense of the 3+1D Universe in which we perceive ourselves to exist, and to measure our perception of change in that Universe.



I was thinking that the measurement of time was initially
to have a sense of when the sun would be swallowed up
by the darkness.

Quote:

Intuitively, I would think not; though our measurement of time may be subject to predictable fluctuations depending on motion or gravity.


Quote:

I have introduced a rational inconsistency. No prizes for spotting it.


so you dont think that time can dilate but you do think
that time may fluctuate depending on motion or gravity.



fluctuate...Dilate...


3/4 inch of dust build up on the moon in 4.527 billion years,LOL and QM is fantasy science.
paul #55537 02/21/16 01:11 PM
Joined: Aug 2010
Posts: 3,570
B
Bill S. Offline OP
Megastar
OP Offline
Megastar
B
Joined: Aug 2010
Posts: 3,570
Quote:
so you dont think that time can dilate but you do think
that time may fluctuate depending on motion or gravity.


Not quite, Paul. Our measurement of time may be subject to fluctuations (or dilation, if you prefer) without time itself changing. That’s not the inconsistency I had in mind.


There never was nothing.
Bill S. #55538 02/21/16 04:33 PM
Joined: Mar 2006
Posts: 4,136
P
Megastar
Offline
Megastar
P
Joined: Mar 2006
Posts: 4,136
this is getting ridiculous , Im using the TOR browser to
browse to the SAGG pages to read and to copy text from
and Im using another browser so that I can paste text into
the reply box and submit the reply.

it wont be long before I give up trying to communicate
on SAGG and will become strictly a reader of the forums.

Quote:
Our measurement of time may be subject to fluctuations (or dilation, if you prefer) without time itself changing.


in other words our measuring equipment that we use to
measure time with may be influenced by motion and gravity
causing the fluctuations of time or dilated time like the
time differences noticed on the mechanical wrist watches of our earlier astronauts.

Im curious though , if you don't believe that time can
fluctuate or dilate then why do you concern yourself with
questioning the existence of time as if time were a physical
object that has physical properties attached to it?

your OP shows that you do question the existence of time
as existing independently from the word time itself.


Quote:
What is a second? That’s an easy one; a second is defined as 9 192 631 770 oscillations of a caesium 133 atom.

That’s all settled, then; or is it?

Does a second have any independent existence, or is it just something that has emerged from arbitrary divisions of years and days?

Compare, for example, a second and a proton. It seems quite reasonable to assume that a proton has an existence that is independent of our measurements. It is probably safe to assume that the diameter of a proton would be about 1.7536 femtometres, whether or not we ever measured, or even thought about it.

Can we say the same for a second? Would a second exist as anything in its own right if it had never been defined? If not, does that tell us anything about the “reality” of time? One thing we know about the second is that it is the SI base unit for the measurement of time. So, for example, if the base unit of time has no independent existence, can time be said to have independent existence?


3/4 inch of dust build up on the moon in 4.527 billion years,LOL and QM is fantasy science.
paul #55542 02/21/16 07:07 PM
Joined: Aug 2010
Posts: 3,570
B
Bill S. Offline OP
Megastar
OP Offline
Megastar
B
Joined: Aug 2010
Posts: 3,570
Quote:
Im curious though , if you don't believe that time can
fluctuate or dilate then why do you concern yourself with
questioning the existence of time as if time were a physical
object that has physical properties attached to it?


Belief in scientific matters is something I try not to hold. I will defend my interpretations as best I can, but if other thinking people hold different views, I always want to know why they think as they do.


There never was nothing.
pokey #55547 02/23/16 04:23 AM
Joined: May 2011
Posts: 2,819
O
Megastar
Offline
Megastar
O
Joined: May 2011
Posts: 2,819
Originally Posted By: pokey
So you’re saying that Albert E said:
If Space = true, Energy and Time = false.

NO you left out a slash option, he joins space and time together into one word "spacetime" and energy is the curvature of that spacetime

If Space = true then Energy & Time can be True/False

Lets give you examples

Photon of light : Space = true, Energy = True, Time = False
Here energy is moving thru space with time stopped in its reference.

Spontaneous decay: Space = true, Energy = False, Time = True
Here a particle breaks into pieces with no interaction, time was running and it decayed for no reason other than time.

The last one became a prediction of his theory
Energy=False and Time=False which is black holes. Energy can't get out of them nor time. Truely weird sitution given to us from the outside they seem to contain enormous energy.

So time and energy are an illusion of space depending where you are positioned. Thats why if you get time or energy zero you just take a different point in spacetime as it means you are looking from a bad reference frame. That is all we do with the photon we look at it from our reference. Similarly when looking from here at an event horizon of a black hole time appears to be stopping so take a position closer to the horizon to fix it.


I believe in "Evil, Bad, Ungodly fantasy science and maths", so I am undoubtedly wrong to you.
Orac #55549 02/23/16 12:12 PM
Joined: Aug 2010
Posts: 3,570
B
Bill S. Offline OP
Megastar
OP Offline
Megastar
B
Joined: Aug 2010
Posts: 3,570
Quote:
Photon of light : Space = true, Energy = True, Time = False
Here energy is moving thru space with time stopped in its reference.


How do you justify ascribing a RF to a massless particle travelling at c?


There never was nothing.
Orac #55550 02/23/16 02:15 PM
Joined: Dec 2010
Posts: 1,858
B
Megastar
Offline
Megastar
B
Joined: Dec 2010
Posts: 1,858
Originally Posted By: Orac
Photon of light : Space = true, Energy = True, Time = False
Here energy is moving thru space with time stopped in its reference.

Are you sure that you mean "in its reference"? I thought that the photon's time was stopped in our reference.

Bill Gill


C is not the speed of light in a vacuum.
C is the universal speed limit.
Bill #55551 02/23/16 11:07 PM
Joined: Aug 2010
Posts: 3,570
B
Bill S. Offline OP
Megastar
OP Offline
Megastar
B
Joined: Aug 2010
Posts: 3,570
Quote:
I thought that the photon's time was stopped in our reference.


Wouldn't that mean that we would never see light move?

It would have to be the photon's RF; if it has one.


There never was nothing.
Bill S. #55552 02/23/16 11:41 PM
Joined: Dec 2010
Posts: 1,858
B
Megastar
Offline
Megastar
B
Joined: Dec 2010
Posts: 1,858
That's where it gets very confusing. I'm not sure, but I really think that the photon's time is stopped in our reference frame. If something is moving with respect to us its clock runs slower in our reference frame and its length gets shorter, again in our reference frame. Since the photon has no mass the mass is not affected. For a photon this may just be one of those problems that appears to a quick view because it is traveling at the speed limit. I guess we will have to wait for Orac to explain it.

Bill


C is not the speed of light in a vacuum.
C is the universal speed limit.
Bill #55553 02/24/16 12:42 AM
Joined: Aug 2007
Posts: 84
P
Member
Offline
Member
P
Joined: Aug 2007
Posts: 84
"Are you sure that you mean "in its reference"? I thought that the photon's time was stopped in our reference."


Most of this gleaned from "physics.stackexchange.com".

Photon’s travel at c from everyone’s point of view. Doesn’t matter how fast the people are traveling. It’s a universal constant (as noted in your byline). So at the speed of light “c”, no time passing is observed. Which means photons don’t experience time at all.

There is no time dilation for a photon at c.

Some hold there is a reference frame at c, called the "null reference frame".

pokey #55554 02/24/16 02:53 AM
Joined: Dec 2010
Posts: 1,858
B
Megastar
Offline
Megastar
B
Joined: Dec 2010
Posts: 1,858
Pokey, can you define which reference frames you are talking about? From your statements I am not sure what reference frames you mean.

Bill Gill


C is not the speed of light in a vacuum.
C is the universal speed limit.
Bill #55555 02/24/16 03:00 AM
Joined: May 2011
Posts: 2,819
O
Megastar
Offline
Megastar
O
Joined: May 2011
Posts: 2,819
Bill G has already worked out your problem Pokey its just reference frame.

If I take a rest frame the use and min/max of the word dilation inverts if I use the speed of light as a reference. You have used the speed of light and thats fine but the dilation will be backward to me describing it from the rest frame.

Lets just use an example without the word dilation.

Right now a photon is leaving the sun.
In YOUR REFERENCE FRAME it will arrive in 8 minutes to earth,
In THE PHOTONS REFERENCE FRAME it will arrive in 0 Seconds.

So time is moving for you, but not the photon that I think we all agree on.

Now to use the word dilation and talk about it we need to know which frame you want to choose the photons or ours.

It might be safer to ue the word Lorentz Factor which uses the rest frame.

Last edited by Orac; 02/24/16 03:05 AM.

I believe in "Evil, Bad, Ungodly fantasy science and maths", so I am undoubtedly wrong to you.
Bill S. #55556 02/24/16 03:10 AM
Joined: May 2011
Posts: 2,819
O
Megastar
Offline
Megastar
O
Joined: May 2011
Posts: 2,819
Originally Posted By: Bill S.

How do you justify ascribing a RF to a massless particle travelling at c?

It's a valid as any other reference frame according to Einstein.

What you want special reference frames now or did you think this was a zero frame restriction I was violating?

It isn't a zero frame because I am not measuring out. Lookup plancks constant it will connect the energy of the photon to its angular momentum and linear momentum. Angular momentum can only work from the frame of the photon itself as its a spin. Yes if I try to measure out to the universe from this frame I will agree its invalid and we place that restriction.

So my reference frame is the point about which the photon spin is defined and I will restrict myself to local reference to that point and the energy described from that point.

Generally in specific situations in problems we can have "bad reference frames" just because they make calculation problematic. There is nothing wrong with the reference frame really it's just not a good choice for particular calculations.

Last edited by Orac; 02/24/16 03:55 AM.

I believe in "Evil, Bad, Ungodly fantasy science and maths", so I am undoubtedly wrong to you.
Orac #55558 02/24/16 10:48 PM
Joined: Aug 2010
Posts: 3,570
B
Bill S. Offline OP
Megastar
OP Offline
Megastar
B
Joined: Aug 2010
Posts: 3,570
Some years ago, when I was trying to come to terms with SR, I reached the conclusion that a photon would not experience time. However, when I tried running this idea past working scientists I found it was not as straightforward as as that. The following is an extract from my notes at the time.

"I talk about the photon being everywhere at once. Quite obviously, in the frame of reference of an observer, the photon travels through time. It takes one second to travel a little less than 300,000 Km. but I am suggesting that in its own frame of reference it does not experience time. This raises a serious point. Special relativity provides for an inertial frame for everything that has mass, but it does not cover massless particles such as the photon. Talking of the photon having a frame of reference is, strictly speaking, not scientific. The photon must always be observed as travelling at “c”. It cannot be at rest relative to anything. Of course, one could argue that it must be at rest relative to itself, but that is not a very productive line of reasoning. Science has not actually produced definite proof that the photon cannot be assigned an inertial frame, but to maintain that it does have one is pure speculation, and maintaining that it does not have one seems to be the generally accepted position. Taking the time dilation equation to its ultimate conclusion may seem a logical thing to do, but it is not supported by special relativity because of the lack of mass of the photon which puts it outside the remit of special relativity. It seems that the best we can say is that we have no way of knowing if photons experience time, or not. Nor do we have any scientifically accredited theory that covers this, nor any way to test the idea experimentally, as no massive object can reach the speed of light. To some extent this all seems to be a shame, because if it could be established that photons did not experience time, then it could be reasoned that the speed of light equated to infinite speed and several mysteries could be solved. Obviously it would still leave a major problem with the tachyon, because we saw the tachyon accelerating away from the speed of light towards infinite speed. However, we might console ourselves with the knowledge that the tachyon is only a theoretical particle".

I assume you have an argument from QM that negates that line of reasoning.


There never was nothing.
Bill S. #55559 02/25/16 01:06 AM
Joined: May 2011
Posts: 2,819
O
Megastar
Offline
Megastar
O
Joined: May 2011
Posts: 2,819
You have already deduced the problem the photon is spinning it clearly has an inertial frame relative to the spin point and yet in your world the spin has two problems time is effectively stopped and the aprticle is massless.

The only conclussion a sane person can reach is we made an error our experiments or the thing is spinning in another dimension not in our world that time is still running in.

I don't see how you think there are any other choices there is an error (which noone has been able to find) or you have another dimension. Please suggest any other solution you feel we can have.

This leads to how we can overlap things in the atomic structure we have an extra dimension.

So specifically in terms of GR I am restricting myself to taking the center point of the spin. We all agree that spin has energy so relative to the spin point it is inertial (the 1st requirement of GR), I have a inertial frame. Next I am only measuring into the extra dimesion(s) that GR does not cover so in no way will anything I calculate violate GR. So in some ways I am doing exactly what you suggested making a self reference frame to the photon.

I would also point out that this treatment of energy and force arising at a single point in your 3D classical world is entirely consistent with how your treat fictional forces like centripetal acceleration which do exactly the same.

So there is a specific solution, I leave GR completely intact and I have extra dimensions. The reality is it doesn't actually even matter if that is the "TRUE ANSWER" it gives you a usable solution which is useful.

Do I really care if the extra dimensions exist or our religious crackpots and layman believe it ... well no not in the slightest. At the very least what I get is a mathematical way to treat physics which doesn't blow up in your face like you have already deduced correctly.

Again I repeat physics job is to be useful not worry about what "feels right" to layman or anybody else and you don't get a vote.

The only consistent way out of your dilema is to introduce an extra dimension and we already know GR does the same trick. So at this point your are at 5 dimensions (3D + time + our extra) and its going to get worse later on. However for now my 5 dimensions will allow me to calculate and predict the correct behaviour of the atom while you are still stuck trying to even remotely work out what to do about things and your physics is in tatters. So which do you think is more useful?

Last edited by Orac; 02/25/16 01:41 AM.

I believe in "Evil, Bad, Ungodly fantasy science and maths", so I am undoubtedly wrong to you.
Bill S. #55561 02/26/16 02:57 AM
Joined: Aug 2010
Posts: 3,570
B
Bill S. Offline OP
Megastar
OP Offline
Megastar
B
Joined: Aug 2010
Posts: 3,570
How does having spin enable an inertial frame to be ascribed to a photon?


There never was nothing.
Bill S. #55570 02/28/16 04:05 PM
Joined: May 2011
Posts: 2,819
O
Megastar
Offline
Megastar
O
Joined: May 2011
Posts: 2,819
Originally Posted By: Bill S.
How does having spin enable an inertial frame to be ascribed to a photon?

You really should be able to do that one yourself ... what is spinning?

Remember you told me the photon was massless yet when it slams into something it imparts energy and we can measure the spin.

If you are still stuck here is the connection
https://en.m.wikipedia.org/wiki/Planck_constant
Please look carefully at the angular momentum and linear momentum derivation for your photon.

What is the more general term for inertia .. you have to see the connection by now.


I believe in "Evil, Bad, Ungodly fantasy science and maths", so I am undoubtedly wrong to you.
Orac #55572 02/28/16 05:48 PM
Joined: Aug 2010
Posts: 3,570
B
Bill S. Offline OP
Megastar
OP Offline
Megastar
B
Joined: Aug 2010
Posts: 3,570
Quote:
You really should be able to do that one yourself ... what is spinning?


Spinning is rotating about an axis. Doesn’t this just shift the problem to defining a RF for the axis?

Quote:
Remember you told me the photon was massless


My understanding is that the photon has no rest mass. However, when in motion it has kinetic energy, which equates to mass and therefore to inertia.

Quote:
when it slams into something it imparts energy and we can measure the spin.


When it slams into something it ceases to exist as a photon.

BTW, I found this, on which you may wish to comment.

http://physics.stackexchange.com/questions/16018/does-a-photon-in-vacuum-have-a-rest-frame

Quote:
In the modern view each particle has one and only one mass defined by the square of it's energy--momentum four vector (which being a Lorentz invariant you can calculate in any inertial frame):
m2≡p2=(E,p⃗ )2=E2−p⃗ 2
For a photon this value is zero. In any frame, and that allows people to reasonably say that the photon has zero mass without needing to define a rest frame for it.
--------------
I agree completely with @dmckee and would only add that for any particle the elapsed time experienced by that particle in it's rest frame is called the proper time and can be calculated (in units where c=1) by any observer as
dτ2=dt2−dx⃗ 2
and for a photon in a vacuum the proper time is always identically 0. So photons do not experience any passage of time so in that sense also, they do not have a rest frame.


There never was nothing.
Bill S. #55574 02/29/16 12:11 AM
Joined: May 2011
Posts: 2,819
O
Megastar
Offline
Megastar
O
Joined: May 2011
Posts: 2,819
Originally Posted By: Bill S.
Spinning is rotating about an axis. Doesn’t this just shift the problem to defining a RF for the axis?

You really aren't getting it are you.

To define something as "spinning" you have to have a reference frame that is the point about which it is spinning. It's a self evident reference frame the same as for the centrifugal case.

What you can't do is use that reference frame outside the thing that is spinning. The thing that is spinning and you avoided answering is energy. It is identical to the centrifugal case or are you saying there is no reference frame for a spinning mass.

Originally Posted By: Bill S
My understanding is that the photon has no rest mass. However, when in motion it has kinetic energy, which equates to mass and therefore to inertia.

Now go and look up the formula for kinetic energy in your classical world it requires a mass.

Also if you applied E=MC2 you would get a mass so we hand wave our way to saying you can't do that either.

Yeah they did some hand waving again and you accepted it, what we really did is said we have this massless thing but it has inertia and energy and you just have to accept it.

What we do via planck's formulation is hand wave our way to having an inertia where there can't be one under classical physics.

Originally Posted By: Bill S
When it slams into something it ceases to exist as a photon.

Come on do you relly expect me to answer that rubbish. So the spin we measured just jumped out of thin air. Simple cause and effect and conservation of energy says things were not there prior to the photon colliding with our detector and they were there after the photon collides WAS ON THE PHOTON BEFORE THE COLLISSION. I only do science and that is the way it breaks I don't discuss philosophy.


Yeah they just said exactly what I told you that the photon energy has a momentum relative to itself and its a valid ereference frame. That can be a reference frame for discussing the energy locally within the spin but you can't use it as a reference frame to the outside world. As they said when you talk of time be careful what excatly you are asking.

This isn't a hard concept to understand and we already told you that depending on what you want to measure you sometimes get bad reference frames because of RELATIVITY.

The energy in a photon most definitely has a relationship to itself as it is SPINNING and it has no relationship to you (as the spin is in a hidden dimension). The layman usually miss that when we talk of spinning it's not there 3D classical version and that is part of the con we can then pull. I always know when they get close to understanding when they start asking I don't understand how the photon can be spinning.

What we do to you layman is hand wave the answer that when the photon slams into something that energy spinning in the hidden dimension becomes measurable in your classical world. So what you thought was a massless particle suddenly has inertial energy which to you literally came out of nowhere and we have to try and fix your classical physics back up.

Now we have laid bare our lie we told you at school surely you see what we did and why.

If you don't want to discuss the hidden dimension I myself don't know any better way than to use this hand wave technic and I use the same trick. I thought you actually wanted to know the real answer now, so we have to look at the lies we had to play.

Your start point is when we say the photon is spinning what do we really mean because it isn't your naive 3D classical thing.

Last edited by Orac; 02/29/16 03:56 AM.

I believe in "Evil, Bad, Ungodly fantasy science and maths", so I am undoubtedly wrong to you.
Orac #55577 02/29/16 06:44 AM
Joined: Aug 2007
Posts: 84
P
Member
Offline
Member
P
Joined: Aug 2007
Posts: 84
["Your start point is when we say the photon is spinning what do we really mean because it isn't your naive 3D classical thing."]


So this “spin” refers to an object acting like it has angular momentum, but without the object actually rotating like it would in classical physics?

pokey #55579 02/29/16 01:32 PM
Joined: May 2011
Posts: 2,819
O
Megastar
Offline
Megastar
O
Joined: May 2011
Posts: 2,819
Originally Posted By: pokey
So this “spin” refers to an object acting like it has angular momentum, but without the object actually rotating like it would in classical physics?

Yes pokey correct the energy is rotating but not in our 3D world. It is somehow spinning hidden behind, under or inside our universe. This is common to all electromagnetic signals of which light belongs to that you require 4 dimensions.

You can only write them as equation of either the electric field E[x,y,z,t] or magnetic field B[x,y,z,t]. The time component is because the EB fields are disappearing in and out of our 3D world. That oscillation occurs regardless of the fact that the signal is travelling at the speed of light and according to us time has stopped.

When we draw the usual 4D representation of the EM wave most layman either haven't run across the concept that time is stopped at c so how does the EB wave know when time to oscillate at as it moves thru space.

Here is the usual image


Do you see the hand wave we do the EB fields are changing in time but time is stopped at c so how are they changing.

The real problem with the drawing is the vertical lines we draw are a lie there they dont exist like that it's just a single line with the EB intensity varying on that single line. If you drew a line varying from black to white in a set length and back again that is what it really looks like. Those arrow represent the intensity and we have taken them into a 3D space which is a lie.

Lets see if I can do ascii art on a single text line

----****@@@@****----****@@@@****-----

The heavier fatter characters represent an increase in either the E or B field whichever we are looking at but its a single line

I find the 3D image above very misleading to people because they think the waveform really is taking up 3D space. The truth is the black line the arrow base is on is the only real thing the vertical arrows just represent intensity to some arbritrary scale of that black line.

Last edited by Orac; 02/29/16 01:43 PM.

I believe in "Evil, Bad, Ungodly fantasy science and maths", so I am undoubtedly wrong to you.
Orac #55580 02/29/16 01:49 PM
Joined: Aug 2010
Posts: 3,570
B
Bill S. Offline OP
Megastar
OP Offline
Megastar
B
Joined: Aug 2010
Posts: 3,570
Quote:
To define something as "spinning" you have to have a reference frame that is the point about which it is spinning.

No problem with that, except that you are also saying that the photon is not actually spinning in the sense that we would understand it in 3D space. Doesn’t that imply that the RF is outside our dimensions?
Quote:
What you can't do is use that reference frame outside the thing that is spinning.

Doesn’t that take us back to “square one”. There’s a RF there somewhere, but we can’t relate it to our 3+1 D.
Quote:
It is somehow spinning hidden behind, under or inside our universe.

This sounds as though it would be difficult to test.


There never was nothing.
Bill S. #55581 03/01/16 12:19 AM
Joined: May 2011
Posts: 2,819
O
Megastar
Offline
Megastar
O
Joined: May 2011
Posts: 2,819
Originally Posted By: Bill S.
No problem with that, except that you are also saying that the photon is not actually spinning in the sense that we would understand it in 3D space. Doesn’t that imply that the RF is outside our dimensions?

Almost it is valid to exactly one point in our 3D space being the centre of the spin and it is thru that one point the energy enters and leaves. It is exactly the same for the centrifugal case.

Quote:
Doesn’t that take us back to “square one”. There’s a RF there somewhere, but we can’t relate it to our 3+1 D.

No as per above it is related thru exactly one point and thru the energy exchange. What you are really saying is you can't relate it to other points in space and that is true. However that situation isn't unique corner a car hard and that energy and force has no meaning to anyone not in the car. The cornering car energy has meaning to you if you crash into it and that is the same for your photon.

I still get the feeling you think reference frames should be good and usable universally and it is not at all unusual for that not to be the case even in your classical world.It's funny if I asked you how a bird can sit on a high voltage power line you would probably recite an answer but you struggle with the same situation involving motion. It's almost as if you feel they should be more connected.
Quote:
This sounds as though it would be difficult to test.

So was the Higgs field smile

If you hadn't realised the detection of GW150914 creates the same problem you need a dimension for space to contract in and out of for the gravitational wave. That was the problem Paul was struggling with into what is space stretching and contracting.

Its funny we say it proved Einstein right but he actually went backward and forward on the gravitational wave thing over his life because it implied the extra dimension in his GR was real, something he struggled with. He wavered throughout his life on whether the extra dimension in his formulation was just part of the mathematics or real. In the end he sort of adopted the shut up and calculate approach.

Last edited by Orac; 03/01/16 12:40 AM.

I believe in "Evil, Bad, Ungodly fantasy science and maths", so I am undoubtedly wrong to you.
Orac #55584 03/01/16 02:53 AM
Joined: Mar 2006
Posts: 4,136
P
Megastar
Offline
Megastar
P
Joined: Mar 2006
Posts: 4,136
I'm going to suggest this as a training aid for you.

perhaps instead of writing this

Quote:
Its funny we say it proved Einstein right but


try something like this


you know its funny because we say it proved Einstein was right
but ...

before you post it , read it , and think about how we talk to others.

this would greatly help us to understand the things you are
telling us.

sometimes the way you write is a little cloudy.


3/4 inch of dust build up on the moon in 4.527 billion years,LOL and QM is fantasy science.
paul #55588 03/01/16 03:54 PM
Joined: May 2011
Posts: 2,819
O
Megastar
Offline
Megastar
O
Joined: May 2011
Posts: 2,819
Thanks for that.

I will try but English is still not the most natural thing for me, even with speach.


I believe in "Evil, Bad, Ungodly fantasy science and maths", so I am undoubtedly wrong to you.
Orac #55590 03/01/16 07:02 PM
Joined: Aug 2010
Posts: 3,570
B
Bill S. Offline OP
Megastar
OP Offline
Megastar
B
Joined: Aug 2010
Posts: 3,570
Quote:
Almost it is valid to exactly one point in our 3D space being the centre of the spin and it is thru that one point the energy enters and leaves. It is exactly the same for the centrifugal case.


I’m trying to get my head round this, so I’m going to break it down.

1. A photon has spin.
2. This is not the same as classical spin.
3. Particle spin causes a quon to behave as though it had angular momentum, but without actually rotating.
4. Although not rotating, a photon has an axis of spin.
5. This axis provides the only point at which contact is made with our 3+1D, such that a RF can be established for the photon.


There never was nothing.
Bill S. #55592 03/02/16 03:44 AM
Joined: May 2011
Posts: 2,819
O
Megastar
Offline
Megastar
O
Joined: May 2011
Posts: 2,819
Originally Posted By: Bill S.
1. A photon has spin.
2. This is not the same as classical spin.

yes

Originally Posted By: Bill S
3. Particle spin causes a quon to behave as though it had angular momentum, but without actually rotating.

We need to put limits on this. It is doing something that when the particle interacts with our classical world it appears to have angular momentum.

You really struggle with the reality thing

Lets do this with throwing a ball off a merry go round which is something far more down to earth you deal with.

See the problem you can't identify what is actually spinning the mathematics works equally. If you measure the forces it does not arise from any physical interaction between the objects but based on the past history of it's movements.

Spin even in your classical world makes no real sense and so in your classical physics you have to introduce it's forces in a totally fictional way. When you corner your car hard you experience forces and motions that make no sense to an observer unless they observe your motions before hand.

So lets take your car cornering hard and I wasn't observing you but suddenly decided to measure. I am advanced race and suddenly teleported into the car to measure and I would go woah where did these forces come from. At the moment I decided to measure the forces make absolutely no sense to me because I am not GOD I can't see what your past motions were. If we had teleportation that would be one of the dangers teleporting into a spinning situation and getting slammed sideways into things.

What you get comfortable with in our classical world is being able to see the past history of things before we interact with them. Generally we only get caught out jumping on or off moving things. Always fun watching people fall over getting on and off escalators, the young ones call it escalator fail.

Originally Posted By: Bill S
4. Although not rotating, a photon has an axis of spin.

It is rotating just not in your classical way that you can view the path of the past motions. Besides being able to see the motion it makes every bit as much sense or as little sense, as your classical spin the forces and energy play out the same way.

So Quantum spin is every bit as real or as imaginary as your classical spin smile

Originally Posted By: Bill S
5. This axis provides the only point at which contact is made with our 3+1D, such that a RF can be established for the photon.

Which is exactly the same for the centripetal case and that frame has no relevance to a stationary observer it relates only thru calculations on energy.

So I guess I should ask is classical spin real or imaginary?
If I can't see the previous motion how do I know if an object is classically spinning?

Last edited by Orac; 03/02/16 03:56 AM.

I believe in "Evil, Bad, Ungodly fantasy science and maths", so I am undoubtedly wrong to you.
Orac #55599 03/02/16 02:43 PM
Joined: Mar 2006
Posts: 4,136
P
Megastar
Offline
Megastar
P
Joined: Mar 2006
Posts: 4,136


both the ball and the merry go round are spinning.

your gif image shows this.

Quote:
Spin even in your classical world makes no real sense and so in your classical physics you have to introduce it's forces in a totally fictional way. When you corner your car hard you experience forces and motions that make no sense to an observer unless they observe your motions before hand.


spin makes perfect sense to my classical world because its
obvious that everything in the cosmos is spinning.

everything!

duhhh.

at the moment that a photon is emitted it inherits its spin
from the spin of its emitted source.

that spin is certainly not claimable by non classic physics
nor is it only calculable by non classic physics.

the ball is rotating 1 revolution for each 1 revolution
of the merry go round.

the side of the ball facing away from the center of the
merry go round is traveling faster than the side of the ball
facing towards the center of the merry go round.

the ball when released is like an unbalanced wheel.

so when the ball is released the ball will continue to spin at the same rpm rate as the merry go round.

what could possibly be so hard to figure out about that?


3/4 inch of dust build up on the moon in 4.527 billion years,LOL and QM is fantasy science.
Orac #55600 03/02/16 03:45 PM
Joined: Aug 2010
Posts: 3,570
B
Bill S. Offline OP
Megastar
OP Offline
Megastar
B
Joined: Aug 2010
Posts: 3,570
Analogies often break down because, at best, they can be only partially analogous to the real thing.

The throwing a ball from a roundabout analogy works only if the spinning frame is a 3D entity. If we were considering a real roundabout, constructed in such a way that we could either spin the roundabout relative to its base, and the wider environment; or keep the roundabout stationary relative to the environment and spin the base, relative to roundabout and environment; there would be no difference in the angular direction, relative to the base, in which the ball would go.

Jus' sayin' smile


There never was nothing.
Bill S. #55601 03/02/16 04:44 PM
Joined: May 2011
Posts: 2,819
O
Megastar
Offline
Megastar
O
Joined: May 2011
Posts: 2,819
Originally Posted By: Bill S.
Analogies often break down because, at best, they can be only partially analogous to the real thing.

I am not sure why you think it is an analogy?

Explain why you don't think it is real smile

Last edited by Orac; 03/02/16 04:46 PM.

I believe in "Evil, Bad, Ungodly fantasy science and maths", so I am undoubtedly wrong to you.
paul #55602 03/02/16 04:59 PM
Joined: May 2011
Posts: 2,819
O
Megastar
Offline
Megastar
O
Joined: May 2011
Posts: 2,819
Paul your science understanding is so bad at times it's like watching a train wreck in slow motion. I never know whether to look away or try and help.
Originally Posted By: paul
at the moment that a photon is emitted it inherits its spin from the spin of its emitted source.

Most of your statements are bad but this one is a train wreck.

I take it you are saying photons have classical spin?

So I have a torch, globe or led makes no difference. So explain to me when the photons spin in the beam that shines out the front relates to the torch.

To extend the problem I have a laser pointer emitting a laser beam describe the photons in the beam spin for me please.

Last edited by Orac; 03/02/16 06:13 PM.

I believe in "Evil, Bad, Ungodly fantasy science and maths", so I am undoubtedly wrong to you.
Orac #55605 03/02/16 08:00 PM
Joined: Aug 2010
Posts: 3,570
B
Bill S. Offline OP
Megastar
OP Offline
Megastar
B
Joined: Aug 2010
Posts: 3,570
Quote:
Explain why you don't think it is real


Throwing a ball from a roundabout is equivalent in every detail to a spinning photon. Somehow I have grave doubts about that.


There never was nothing.
Orac #55607 03/03/16 02:26 AM
Joined: Mar 2006
Posts: 4,136
P
Megastar
Offline
Megastar
P
Joined: Mar 2006
Posts: 4,136
Quote:
I take it you are saying photons have classical spin?


of course they do.

Quote:
So I have a torch, globe or led makes no difference. So explain to me when the photons spin in the beam that shines out the front relates to the torch.


when the photon leaves the light source.

Quote:
To extend the problem I have a laser pointer emitting a laser beam describe the photons in the beam spin for me please.


the photons in the beam have inherited the motion of the
light source ( the laser ) as they were emitted.

and they will continue to have that inherited spin until
some outside influence acts against the spin like the
influence from a magnetic field or a gravity field
that causes its spin to change.

if you don't believe that a photon inherits the spin of
the light source then perhaps you should explain why it
would not.

lets put a laser pointer in a drill press and turn the
laser pointer on , then turn the drill press on at its highest
speed... for example lets say the drill press is rotating
the laser pointer at 10,000 rpm.

do you seriously believe that the light being emitted
from the laser pointer will not inherit the spin of the
laser.

if so please tell me why the emitted photons would not be spinning at the same rpm as the laser pointer.












3/4 inch of dust build up on the moon in 4.527 billion years,LOL and QM is fantasy science.
paul #55608 03/03/16 03:01 AM
Joined: May 2011
Posts: 2,819
O
Megastar
Offline
Megastar
O
Joined: May 2011
Posts: 2,819
Originally Posted By: paul
when the photon leaves the light source.

Originally Posted By: paul
the photons in the beam have inherited the motion of the light source ( the laser ) as they were emitted.

But what in the light source the atoms orientation and spin, the actual macro spin of the torch. As I said I can't really understand what you think your spin relates too.

Originally Posted By: paul
and they will continue to have that inherited spin until some outside influence acts against the spin like the influence from a magnetic field or a gravity field that causes its spin to change.

Say what ... can you give me an example of an electric or magnetic field deflecting light?

Originally Posted By: paul
if you don't believe that a photon inherits the spin of the light source then perhaps you should explain why it
would not.

Ummm because photon spins aren't classical at all

Originally Posted By: paul
lets put a laser pointer in a drill press and turn the laser pointer on , then turn the drill press on at its highest speed... for example lets say the drill press is rotating the laser pointer at 10,000 rpm.

Spinning the source does nothing and it's easy to test and directly conflicts with your idea.

Originally Posted By: paul
do you seriously believe that the light being emitted from the laser pointer will not inherit the spin of the
laser.

Not only do I believe it you can show it by countless experiments. Light from quasars and pulsars from space which are spinning unbelievably fast have no special super fast Paul spin on them.

Originally Posted By: paul
if so please tell me why the emitted photons would not be spinning at the same rpm as the laser pointer.

No point me explaining it you always make your own physics up and you already know you won't accept it.

Lets just give you two words and a fact you may decide to search. The words are helicity and chirality. The fact is for massless particles the helicity always equals the chirality which it does not for particles with mass. I can't help beyond that because I am not sure how helicity and chirality works in your physics.

Last edited by Orac; 03/03/16 03:37 AM.

I believe in "Evil, Bad, Ungodly fantasy science and maths", so I am undoubtedly wrong to you.
Bill S. #55609 03/03/16 03:32 AM
Joined: May 2011
Posts: 2,819
O
Megastar
Offline
Megastar
O
Joined: May 2011
Posts: 2,819
Originally Posted By: Bill S.
Throwing a ball from a roundabout is equivalent in every detail to a spinning photon. Somehow I have grave doubts about that.

Well they both involve spin and I can't give you an analogue of spin. Something either spins or it doesn't.

The roundabout example shows the relationship of mass movement with spin. So if I have something I can't or haven't seen the motion of I can identify spin if we can understand the motions.

Lots of sports people like pool or billiards player, some soccer players (Bend it like Beckham), most tennis players, some baseball pitchers and many layman know how to use spin because it behaves the same in all physical situations.

You can even write the equations of the sport
https://www.grc.nasa.gov/www/k-12/airplane/straj.html => Bending a soccer ball.

Spinning has no analogs and it simply has equations of what happens whatever the situation. So if I have something I say is spinning it might be important to check it behaves the same.

Last edited by Orac; 03/03/16 03:39 AM.

I believe in "Evil, Bad, Ungodly fantasy science and maths", so I am undoubtedly wrong to you.
paul #55611 03/03/16 02:20 PM
Joined: Aug 2010
Posts: 3,570
B
Bill S. Offline OP
Megastar
OP Offline
Megastar
B
Joined: Aug 2010
Posts: 3,570
Originally Posted By: Paul
lets put a laser pointer in a drill press and turn the
laser pointer on , then turn the drill press on at its highest
speed... for example lets say the drill press is rotating
the laser pointer at 10,000 rpm.


I’m trying to get my head round this.

1. The laser pointer is rotating at 10,000rpm.
2. Does the emerging beam rotate at 10,000rpm?
3. Is this rotation imparted by the rotation of the emitting “element”?
4. Is this rotation imparted by “friction” between the laser lens and the light?
5. Wouldn’t rotation of the beam impart helical movement to the photons, rather than causing each photon to spin, but maintain a straight course?
6. If 5 is correct, would you see this as an example of photons travelling faster than “c”?


There never was nothing.
Bill S. #55612 03/03/16 02:38 PM
Joined: Aug 2010
Posts: 3,570
B
Bill S. Offline OP
Megastar
OP Offline
Megastar
B
Joined: Aug 2010
Posts: 3,570
Returning to a previously mentioned inconsistency:

Originally Posted By: post #55533


Where do I stand on the question of the existence, or non-existence, of time?

Intuitively, I think of time as something we conceived to enable us to make sense of the 3+1D Universe in which we perceive ourselves to exist, and to measure our perception of change in that Universe.

Can time dilate?

Intuitively, I would think not; though our measurement of time may be subject to predictable fluctuations depending on motion or gravity.

At this point, I stop because I have introduced a rational inconsistency. No prizes for spotting it.


If time can be measured it must be something. This is inconsistent with my previous intuitive thought.


There never was nothing.
Bill S. #55613 03/03/16 04:22 PM
Joined: May 2011
Posts: 2,819
O
Megastar
Offline
Megastar
O
Joined: May 2011
Posts: 2,819
I wasn't expecting you to have trouble with this one Bill S.

Originally Posted By: Bill S.
1. The laser pointer is rotating at 10,000rpm.
2. Does the emerging beam rotate at 10,000rpm?

Nope it doesn't it holds no relationship to your classical world .. EB fields don't see Bill S smile

I won't lie to you and give you the usual hand wave and tell you the photon is a point and you can't spin a point laugh

The extension of the exercise is if you run forward with the torch or laser pointer the light beam doesn't leave faster than c either. Nor if you run backwards do the photons leave the torch slower.

I am going to do a hand wave for the moment and say in general you can't control the emission rotation it is entirely probabilistic. So I am going to introduce a new term called Spontaneous Emission of a photon from an atom and tell you the orientation is random under that condition.

Originally Posted By: Bill S.
3. Is this rotation imparted by the rotation of the emitting “element”?

This is the hand wave if I prepare a special form of photons I can stimulate an emission from an atom. The emitted photon will inherit the spin characteristics of the photon that stimulated it and we sometimes do this in lasers to produced a polarized laser beam.

So when you said "element" you were probably thinking classical world stuff but no we do some trickery with QM to control the spin.

Originally Posted By: Bill S.
4. Is this rotation imparted by “friction” between the laser lens and the light?

Spinning the lens has a funny effect it will slow the light down and can shift the photon slightly but it will not impart rotation to the photon. Even if you make the lens long like a cylinder. The process is called photon drag.

http://physicsworld.com/cws/article/news/2011/jul/05/rotating-cylinder-puts-a-new-spin-on-slow-light

Originally Posted By: Bill S.
5. Wouldn’t rotation of the beam impart helical movement to the photons, rather than causing each photon to spin, but maintain a straight course?

6. If 5 is correct, would you see this as an example of photons travelling faster than “c”?

5 doesn't happen the emission process doesn't see classical movements and 6 is correct if it could happen.

I will give you the only way you can control photon spin classically its called a polarization and you see it a lot for layman in glasses. So you actually filter out all the light that doesn't match the orientation you desire at the cost that the light emerging is diminished in intensity because you select only the photons with the spin you want.

Even in a laser beam the photons would leave with a random spin unless you put a polarizer in the system in one of a number of forms. However usually because of the design of the partial mirrors at best you usually get partial polarization out of an unpolarized laser. I doubt you can get a truely random one like normal incandescent process because of the stimulated emission.

Finally fixing the hand wave there is a QM effect predicted and experimentally verified called Delbrück scattering. It's fleeting but it might be possible to extend the deflection with lots of QM trickery.

Excluding Laser, Masers and some QM stuff and holding to your classical world we can make this statement.
Originally Posted By: Max Planck
Most materials emit photons from their surface in a random direction and phase. If the energy source is heat (like an incandescent light), the frequency is also randomly drawn from a probability distribution which depends on the material's temperature.

Finally if you want to see the full story this wiki page is one of the best going around but OAM will probably be confusing to you and why not all spins interact with matter the same way (https://en.wikipedia.org/wiki/Angular_momentum_of_light).

Last edited by Orac; 03/03/16 05:48 PM.

I believe in "Evil, Bad, Ungodly fantasy science and maths", so I am undoubtedly wrong to you.
Orac #55614 03/03/16 04:27 PM
Joined: Mar 2006
Posts: 4,136
P
Megastar
Offline
Megastar
P
Joined: Mar 2006
Posts: 4,136
Quote:
But what in the light source the atoms orientation and spin, the actual macro spin of the torch. As I said I can't really understand what you think your spin relates too.


I'm not sure what you were trying to communicate when
you wrote the above orac.

Quote:
Say what ... can you give me an example of an electric or magnetic field deflecting light?


Im going to guess that the word you meant to use was
attract vs deflecting.

try thinking about a cathode ray tube as in a old style
tv tube ...


Quote:
Electromagnetic radiation (EM radiation or EMR) is the radiant energy released by certain electromagnetic processes. Visible light is one type of electromagnetic radiation; other familiar forms are invisible electromagnetic radiations, such as radio waves, infrared light and X rays.

Classically, electromagnetic radiation consists of electromagnetic waves, which are synchronized oscillations of electric and magnetic fields that propagate at the speed of light through a vacuum.


a beam of laser light is an electromagnetic wave.

a electric magnetic field can bend and spin a electromagnetic wave.




a magnet can bend a laser beam.



Quote:
Ummm because photon spins aren't classical at all


that's classic.

Quote:
Spinning the source does nothing and it's easy to test and directly conflicts with your idea.


BS

lets see a link to the test you speak of... LOL laugh

or should I simply take your word for it...


3/4 inch of dust build up on the moon in 4.527 billion years,LOL and QM is fantasy science.
paul #55615 03/03/16 04:36 PM
Joined: May 2011
Posts: 2,819
O
Megastar
Offline
Megastar
O
Joined: May 2011
Posts: 2,819
Originally Posted By: paul
try thinking about a cathode ray tube as in a old style
tv tube ...

Paul ... that's a trainwreck.

You are talking about an electron google cathode ray tube.
Electrons have charge you can deflect them via the charge.

Light photons have no charge and no mass .. NEWSFLASH NOTHING DEFLECTS THEM.

In GR you can lense them around very strong gravity sources but you don't believe in GR so no issue here.

You can only classically DIFFRACT photons which your videos show and that requires a medium. Controlling the medium with a magnet or a electricity is not the same thing.

Now show me a video of light beam being deflected in a vacuum please .... as you say the have electric and magnetic fields so show me them being deflected like a CRT.

Quote:
ets see a link to the test you speak of... LOL laugh

http://physicsworld.com/cws/article/news/2011/jul/05/rotating-cylinder-puts-a-new-spin-on-slow-light

All that happens is you will slow the light ... oopppsie Paul physics failed again.

The only way to control photon spin classically is with polarizers (https://en.wikipedia.org/wiki/Polarizer) .. read it.

As you have reached my stupidity limit in this discussion I am pretty well done here .. please don't direct questions back to me I am not interested in Paul physics it's in the same league as Marosz physics. I don't discuss pseudoscience junk please take this up with others. I don't need to practice my English that bad that I am willing to discuss lunatic rantings.

Last edited by Orac; 03/03/16 05:15 PM.

I believe in "Evil, Bad, Ungodly fantasy science and maths", so I am undoubtedly wrong to you.
Bill S. #55616 03/03/16 05:14 PM
Joined: Mar 2006
Posts: 4,136
P
Megastar
Offline
Megastar
P
Joined: Mar 2006
Posts: 4,136
Quote:
I’m trying to get my head round this.

Quote:
1. The laser pointer is rotating at 10,000rpm.

yes because the laser pointer is inserted into the drill
press as if it were a drill bit.

Quote:
2. Does the emerging beam rotate at 10,000rpm?


the entire beam would rotate


Quote:
3. Is this rotation imparted by the rotation of the emitting “element”?


the emitting element is rotating so any photons that it
emits will also rotate.


Quote:
4. Is this rotation imparted by “friction” between the laser lens and the light?



the laser lens is rotating at the same rpm as the laser beam
so the light passing through the lens would feel the same
resistance to movement as it would if the laser were not rotating.


Quote:
5. Wouldn’t rotation of the beam impart helical movement to the photons, rather than causing each photon to spin, but maintain a straight course?


yes the photons would have a helical rotation but all of
the emitted photons would be spinning in unison and travel
in a straight line , if the laser pointer only emitted a
single photon then the single photon would spin at 10,000 rpm
and there wouldnt be a formation of a helical shape of the
resultant light beam but since the laser beam is much wider than the width of a single photon then yes the beam would be
helical.


Quote:
6. If 5 is correct, would you see this as an example of photons travelling faster than “c”?


the photons at the outside edge of the beam would be
traveling faster than the photons at the center of the
rotating beam because they are further from the center
of the beam , but they would not travel faster linearly
than the photons at the center of the beam.

so even though the overall photon beam would be traveling
at c if in a vacuum the photons at the edge of the beam
would be traveling slightly faster than c due to the
combination of linear and angular movement of the photons
but not much faster.

unless the photons have decided to adhere to the Einstein
speed limit.





3/4 inch of dust build up on the moon in 4.527 billion years,LOL and QM is fantasy science.
paul #55617 03/03/16 06:14 PM
Joined: Aug 2010
Posts: 3,570
B
Bill S. Offline OP
Megastar
OP Offline
Megastar
B
Joined: Aug 2010
Posts: 3,570
Quote:
unless the photons have decided to adhere to the Einstein
speed limit.


I guess the choice between faster than c, and time dilation is an easy one for you.

I sometimes wonder if I should regret leaving (most) certainties behind many years ago. Life was simpler then, but not as interesting. smile


There never was nothing.
Bill S. #55620 03/03/16 10:49 PM
Joined: Mar 2006
Posts: 4,136
P
Megastar
Offline
Megastar
P
Joined: Mar 2006
Posts: 4,136
Quote:
I guess the choice between faster than c, and time dilation is an easy one for you.


yep

Quote:
I sometimes wonder if I should regret leaving (most) certainties behind many years ago. Life was simpler then, but not as interesting.


I'm sure you remember all the talk about allowing science
to be more interesting to attract more students into the
sciences ... they did , it happened , and now they are stuck
with it because of the money that the interesting science
makes for those who make the money from it...

I'm sure they have people who understand how things will
actually happen as a check to ensure the gullible hordes
don't make any actual disasters occur as they are playing
in the fantasy realm of modern science.


3/4 inch of dust build up on the moon in 4.527 billion years,LOL and QM is fantasy science.
paul #55621 03/04/16 04:29 AM
Joined: May 2011
Posts: 2,819
O
Megastar
Offline
Megastar
O
Joined: May 2011
Posts: 2,819
Paul, Science is bad evil money grabbers ... post count 2 since discussing this continued repetition smile

We got the message of what you believed in every post before now.

You are adding nothing new to discussion and just repeating the same spam slogans.

If all you want to do is spam the same message then please make your own thread and spam away, the same situation is imposed on Marosz. You can say whatever you like just please don't pollute every thread and make it difficult for people to actually discuss details.

Last edited by Orac; 03/04/16 04:43 AM.

I believe in "Evil, Bad, Ungodly fantasy science and maths", so I am undoubtedly wrong to you.
Orac #55624 03/04/16 12:56 PM
Joined: Aug 2010
Posts: 3,570
B
Bill S. Offline OP
Megastar
OP Offline
Megastar
B
Joined: Aug 2010
Posts: 3,570
Quote:

You are adding nothing new to discussion and just repeating the same spam slogans.


I have to take some responsibility there, I did hand Paul that one, on a plate; mea culpa!


There never was nothing.
Orac #55625 03/04/16 04:37 PM
Joined: Mar 2006
Posts: 4,136
P
Megastar
Offline
Megastar
P
Joined: Mar 2006
Posts: 4,136
are you really that dense orac?

here is the question you posted to me.

Quote:
Say what ... can you give me an example of an electric or magnetic field deflecting light?


I showed you several videos where light is bent / deflected
using both an electric field and a magnetic field.

in the below video you can SEE the light if you couldn't
see the light then it would not be light now would it.

you can SEE the light bending / deflecting.

are you now going to say that the light SEEN in the video
and SEEN bending / deflecting is not actually light?



if the light SEEN in the video is not light then
please tell me why you can see it...

also

the brightness difference of the light showing through the electric coil in the other video I posted is not the
result of light passing through a medium it is the result
of light passing through a magnetic field that causes the change in brightness that is shown in the video.

also

the link that you posted about the ruby spin medium
could not be used to show the effects or results of a
spinning laser light like I have posted as the light
in the experiment is a image that is projected onto
a spinning ruby medium ... theres a big difference
there.

it does however show that light can be bent inside
a medium without using gravity or a electric field
or a magnet.

before you post your garbage that you find in your
fantasy trash cans you might at least try to see if
anything that you ever post actually relates to the
things that you are posting about.

your entire argument is void concerning the long
winded pat on your back with your hand attempt to
debunk my post on the spinning laser light but don't
start crying yet there is a easy way to check it.

if your BS science is right about the speed of light having
a set speed limit then the photons at the edge of the
spinning laser beam will arrive at a target after the
photons at the center of a spinning laser beam.

because nothing can travel faster than the speed of
light not even light itself, right Einstein!

if your BS science is right the laser beam would
form a cone shape !

with the tip of the cone being the photons in the
center of the laser beam.

I'm going to say that the photons will all arrive at
the same time.




3/4 inch of dust build up on the moon in 4.527 billion years,LOL and QM is fantasy science.
paul #55626 03/04/16 10:04 PM
Joined: Aug 2010
Posts: 3,570
B
Bill S. Offline OP
Megastar
OP Offline
Megastar
B
Joined: Aug 2010
Posts: 3,570
Quote:
if your BS science is right about the speed of light having a set speed limit then the photons at the edge of the
spinning laser beam will arrive at a target after the photons at the center of a spinning laser beam.


Paul, that's only half of the "BS science". The spiralling photons would "experience" time dilation, so they should arrive at the same time as the central photons.


There never was nothing.
Bill S. #55628 03/05/16 02:07 AM
Joined: Mar 2006
Posts: 4,136
P
Megastar
Offline
Megastar
P
Joined: Mar 2006
Posts: 4,136
Quote:
Paul, that's only half of the "BS science". The spiralling photons would "experience" time dilation, so they should arrive at the same time as the central photons.


LOL

I didn't think of that because I don't believe that time
can dilate , but that shouldn't be a real problem as the
time dilation couldn't possibly cause the photons to
arrive at the same time if different laser rotation speeds
and if needed wider laser beams are used ...

this way if they try to claim that time dilation causes
the photons to arrive at the same time then through the
use of varying laser rotation speeds and laser width
a conclusion that the photons are experiencing time
dilation could not be found to be valid unless they prop
up the claim with even more false math that would reflect
the variations in speed and laser width.

I would love to see the experiment myself.

but modern science wouldn't carry out an experiment
that would lessen the popularity of the magic show.



3/4 inch of dust build up on the moon in 4.527 billion years,LOL and QM is fantasy science.
paul #55629 03/05/16 03:22 AM
Joined: May 2011
Posts: 2,819
O
Megastar
Offline
Megastar
O
Joined: May 2011
Posts: 2,819
Paul you call me dense the title of your video is deflection of a electron beam.

You like Marosz fail even the basics, an electron is not a photon, and your whole post requires no discussion.

Now we are done here fool I don't do Marosz like stupidity discussions. Please talk to someone else about it if you really are so intellectually challenged you don't get what is happening, I am treating this as a religious crackpot trolling because it is beyond stupid.

Now on my English lesson whats a less insulting way of saying "beyond stupid" that is when something is so stupid it can only be a troll, deliberate misinformation or a blatant lie.

Last edited by Orac; 03/05/16 03:48 AM.

I believe in "Evil, Bad, Ungodly fantasy science and maths", so I am undoubtedly wrong to you.
Bill S. #55630 03/05/16 02:12 PM
Joined: Aug 2010
Posts: 3,570
B
Bill S. Offline OP
Megastar
OP Offline
Megastar
B
Joined: Aug 2010
Posts: 3,570
Originally Posted By: Orac
Now on my English lesson whats a less insulting way of saying "beyond stupid"


Unconscionably naïve ? smile


There never was nothing.
Orac #55631 03/05/16 04:08 PM
Joined: Mar 2006
Posts: 4,136
P
Megastar
Offline
Megastar
P
Joined: Mar 2006
Posts: 4,136
Quote:
Paul you call me dense the title of your video is deflection of a electron beam.


it doesn't matter what the title of the video was.

its clear that the light or photons are being bent
to the same angle as the electron beam.

looking at the beam on a macroscopic scale the individual
electrons that are emitted from the electron gun are extremely
far apart and on our scale they are apx as far away from each other as a grain of sand in the center of a football stadium
is away from a grain of sand sitting on the top row of the
stadium.

if the photons are not being bent by the electric field
then they would not follow the same path as the electrons
now would they.

your probability trash couldn't even solve for why the
photons are capable of exciting any other electron after it
is emitted from a electron much less how it could accomplish
the needed chain of excitation and emission all the way to the end of the electron beam , so unless you can show how the
photons are following the curve of the electrons then the
only way that the photons could possibly be curving WITH
the electrons is if they are being directed along the same curve by the field.


3/4 inch of dust build up on the moon in 4.527 billion years,LOL and QM is fantasy science.
paul #55632 03/06/16 05:08 AM
Joined: May 2011
Posts: 2,819
O
Megastar
Offline
Megastar
O
Joined: May 2011
Posts: 2,819
Paul you are either, as Bill S says unconscionably naïve, or just plain ignorant and uneducated. The design of the experiment is deliberate, the fact you unlike most children (the target audience of the experiment) don't understand it is not my concern. As I have stated the title of the video is accurate and will leave it at that and I have nothing to add.

You like Marosz are free to believe whatever you like and I really don't care and why should I. When you have something not so unconscionably naïve I may choose to comment. I do the same thing to Marosz ignoring his ridiculous posts and children drawings which aren't worthy of commenting, although often I get a laugh at as I did with yours. Your or any other laymans opinion has no bearing on anything, and has no significance except to you. You always come across to me as if I should care what you believe and it is important, which I assume is your religious background where converting unbelievers is important.

For now you are on the ignore list like Marosz. You, at least when you are not doing anti-science rants, can construct a reasonable sentence so I will simply wait for the subject to return to something more intelligent that justifies a comment.

Last edited by Orac; 03/06/16 06:55 AM.

I believe in "Evil, Bad, Ungodly fantasy science and maths", so I am undoubtedly wrong to you.
Orac #55633 03/06/16 04:29 PM
Joined: Mar 2006
Posts: 4,136
P
Megastar
Offline
Megastar
P
Joined: Mar 2006
Posts: 4,136
Quote:
as Bill S says unconscionably naïve


from what I read you asked a question about how to word
something in a nicer way and Bill S told you how to do it
he wasn't saying that (I) was unconscionably naïve just
to set the record straight.

I would still like to have your reply as to why you can see
the electron beam in the video I posted.

1) the electron stream being emitted from an electron gun
passes through a focusing tube that directs the electrons
through a narrow orifice at the end of the electron gun.

2) the only light that could possibly exit the end of the
electron gun is the light that is generated from the heating of the cathode way in the back of the electron gun.

3) any light that might exit the end of the electron gun
would follow a straight line not a curved path and certainly not the same path as the electron beam unless the light is
also being bent by the electric fields.

4) in the video you can see the light that is traveling
in a straight line as the electron beam is moved.
the light generated from the heat of the cathode continues
in a straight line while the LIGHT of the electron beam
is being curved away with the electron beam.

5) electrons are invisible ! so when you see the LIGHT
of a electron beam you are not seeing the electrons you
are seeing photons of light.




just because your brain doesn't want to accept things for
what they are due to the washing it has undergone does not
change the fact that those things are actually occurring
in reality.

is it true that you don't have the ability to answer the
questions because your degree of physics knowledge is
really very very low.

you never do want to answer any valid questions concerning
your BS science and that is one of the main reasons that I
fully believe that your BS science is truly BS science.

your constant religious rants have no meaning coming from you
or anyone else on this science forum so why don't you attempt
to be slightly scientific and stick to the discussion and
try and avoid the non scientific jargon that you seem to
know more about than science according to the answers that
you never give when asked.



3/4 inch of dust build up on the moon in 4.527 billion years,LOL and QM is fantasy science.
paul #55634 03/06/16 10:10 PM
Joined: Aug 2010
Posts: 3,570
B
Bill S. Offline OP
Megastar
OP Offline
Megastar
B
Joined: Aug 2010
Posts: 3,570
A thought (possibly naïve). In the video we are viewing the demonstration from about 90 degrees to the propagation direction of the beam. Unless there is a fairly dense atmosphere in the chamber, why is the beam visible?


There never was nothing.
Bill S. #55635 03/07/16 12:28 AM
Joined: Mar 2006
Posts: 4,136
P
Megastar
Offline
Megastar
P
Joined: Mar 2006
Posts: 4,136
Quote:
Unless there is a fairly dense atmosphere in the chamber, why is the beam visible?


heres a video of a crt tube that has argon gas in the tube
which makes the electron stream visible.



and link to a electron gun that has high vacuum in the
tube.

http://www.kimphys.com/electron_guns/catalog_PDFs/Gun_intro_prelim.pdf

the tube in the above electron gun has a vacuum pressure
of apx 1 x 10^-7 torr so theres really no atmosphere
as 1 torr is .0013 atm

light propagates in all directions so no matter what angle
you view the tube from you would see the light beam
generated by the heat of the cathode and the light beam
of the emitted electron stream.

your probably thinking that because light propagates in
all directions the light photons from the cathode are
exciting the electrons in the electron stream and then
re-emitting them and the light photons
strike the next electron further down in the stream
without direction of any sort so the propagation of light
along the electron stream is due only to the electrons being
there and the light propagating in all directions.

if that were the case then knowing that the atoms
that form the argon gas molecules also have electrons
should mean that the light that is seen would not follow
any path at all because the crt tube is filled with electrons
even before the beam is started.

the only light that that we can see with our eyes is
the photons of visible light and the video filmed
the light photons of visible light.

you need to think about a focused light beam and why
does light from a laser beam travel in a straight line
and is not vectored away from that straight line by
the electrons in the air that we breath !!!

why does a laser beam travel in a straight line.

a focused electron beam is similar to a focused laser beam.

and then ask yourself how could the electron beam be
emitting visible light photons that are directed
in the same path as the electron stream in the same
manner as the beam of a laser.





3/4 inch of dust build up on the moon in 4.527 billion years,LOL and QM is fantasy science.
Orac #55636 03/07/16 01:05 AM
Joined: Mar 2006
Posts: 4,136
P
Megastar
Offline
Megastar
P
Joined: Mar 2006
Posts: 4,136
Originally Posted By: orac
Light photons have no charge and no mass .. NEWSFLASH NOTHING DEFLECTS THEM.


that's strange because we haven't been able to see inside
a black hole yet...

and GR has nothing to do with reality , the reality is
that gravity existed long before Einstein or his relatives did.

perhaps the black holes don't yet understand that they should
be allowing light to escape them because nothing can deflect light photons. laugh

if a photon has no mass or charge then how do the black holes
swallow up all them photons of light and keep them from being seen?

NEWSFLASH I THINK YOUR WRONG AGAIN !!!!!







3/4 inch of dust build up on the moon in 4.527 billion years,LOL and QM is fantasy science.
paul #55637 03/07/16 02:20 AM
Joined: May 2011
Posts: 2,819
O
Megastar
Offline
Megastar
O
Joined: May 2011
Posts: 2,819
Paul, you clearly like Marosz have a untreated mental condition that precludes you understanding written statements.

Lets try again Paul I am not remotely interested in this discussion, why direct questions at me. The fact you can't work it out is not my problem, and I don't care to set the record straight because you have gone to religious crackpot land. This is like the dinosaurs on the ark just to silly for me to discuss.

Besides it is more fun watching you make a fool of yourself smile

Last edited by Orac; 03/07/16 02:31 AM.

I believe in "Evil, Bad, Ungodly fantasy science and maths", so I am undoubtedly wrong to you.
Bill S. #55638 03/07/16 02:30 AM
Joined: May 2011
Posts: 2,819
O
Megastar
Offline
Megastar
O
Joined: May 2011
Posts: 2,819
Originally Posted By: Bill S.
Unless there is a fairly dense atmosphere in the chamber, why is the beam visible?

Seriously Bill S you don't get it?

The beam is visible BY DESIGN you place a gas in that reacts with the energy range of the electrons. The light will spread randomly in all directions from the collision that absorbs the electron. So you lose one electron and gain a photon emission from that point. It's very straight forward.

When I do it for kiddies I use a single electron source with a detector at the electron exit point that emits a sound and a photo multiplier to detect the flash. You either get the sound (the electron got thru with no collision) or you get a flash (collision happened). You never get both and most kiddies instantly get it.

The fact you can see the beam is basically an inefficiency of electrons being absorbed before striking the screen for display which is the purpose of a CRT.

Anyhow that about all I have to say on the matter and I don't intend to debate it because this is like kiddie level physics. This is not something I can take seriously and it smacks of Marosz like discussions completely pointless and very stupid.

Originally Posted By: https://en.wikipedia.org/wiki/Cathode_ray
Cathode rays are invisible, but their presence was first detected in early vacuum tubes when they struck the glass wall of the tube, exciting the atoms of the glass and causing them to emit light, a glow called fluorescence

If you can see the beam it's by design because thats what we wanted to do by using a specific gas ... now do you really want to continue to discuss stupidity? I was getting a really good laugh, it should have taken anyone around 10 sec search to find the beam is normally invisible, except for with the Science a GoGo forum and some of it's whacky inhabitants.

Trainwrecks are can be interesting to watch smile

Last edited by Orac; 03/07/16 06:27 AM.

I believe in "Evil, Bad, Ungodly fantasy science and maths", so I am undoubtedly wrong to you.
Orac #55639 03/07/16 12:45 PM
Joined: Aug 2010
Posts: 3,570
B
Bill S. Offline OP
Megastar
OP Offline
Megastar
B
Joined: Aug 2010
Posts: 3,570
Quote:
The beam is visible BY DESIGN you place a gas in that reacts with the energy range of the electrons


In other words; the photons are not emitted from the electron source, they result from reaction between electrons and gas. so they must curve with the electron beam.


There never was nothing.
Bill S. #55641 03/07/16 02:28 PM
Joined: May 2011
Posts: 2,819
O
Megastar
Offline
Megastar
O
Joined: May 2011
Posts: 2,819
Originally Posted By: Bill S.
In other words; the photons are not emitted from the electron source, they result from reaction between electrons and gas. so they must curve with the electron beam.

Bingo and all I did was be mean and withhold that the beam is normally invisible smile

It's actually quite hard to make the beam show really brightly but there is a cheat you can do and put an RF exciter or Tesla slayer circuit on the electron gun to artificially ignite the gas along the path of the beam as well.

Last edited by Orac; 03/07/16 03:00 PM.

I believe in "Evil, Bad, Ungodly fantasy science and maths", so I am undoubtedly wrong to you.
Orac #55642 03/07/16 02:59 PM
Joined: Mar 2006
Posts: 4,136
P
Megastar
Offline
Megastar
P
Joined: Mar 2006
Posts: 4,136
Quote:
it should have taken anyone around 10 sec search to find the beam is normally invisible, except for with the Science a GoGo forum and some of it's whacky inhabitants.


Bill S already knew that electrons are normally invisible
the reason I posted that electrons are invisible was for
your benefit ...

I believe that Bill S was asking WHY we could see the
light (the process)
and not whether or not the tube was designed so
that we could see the LIGHT.

several days go by after something is pointed out then you
jump in and start boasting about it as if you already knew it.

that's how you operate isn't it.

and then you say that you were waiting to see if someone
could figure it out first ... LOL

you remind me of Microsoft programmers who rely on
the user community to fix the problems in their operating systems then they incorporate those fixes into the OS as updates.

but you go one step further by laying claim on those fixes.

Quote:
The light will spread randomly in all directions from the collision that absorbs the electron.


the light will spread in all directions , you can keep
your BS randomness.

how can light spread randomly in all directions.

Quote:
collision that absorbs the electron. So you lose one electron and gain a photon emission from that point. It's very straight forward.


could you explain that process in detail?

1) what does the electron collide with?

2) why is an electron lost?

3) where does the lost electron go?

3) why is a photon gained?

its very straight forward so you should have no troubles
explaining the entire process involved from electron
collision all the way through to photon emission.

Quote:
When I do it for kiddies


when you do it for kiddies do you explain the process
or do you just allow them to view the show?


3/4 inch of dust build up on the moon in 4.527 billion years,LOL and QM is fantasy science.
paul #55643 03/07/16 03:06 PM
Joined: May 2011
Posts: 2,819
O
Megastar
Offline
Megastar
O
Joined: May 2011
Posts: 2,819
No Paul I am not here to convert you and I won't waste my time explaining things to some religious crackpot, who isn't remotely interested in understanding.

Get it through your head I really don't care what you believe, go and make up more Paul physics. You some how think I have to do this and I have to explain that ... sorry I don't and I don't discuss things this basic.

It's like Dinosaurs on the Ark I don't try and convert you I just find it funny and mildly amusing.

It you want to understand the process go read yourself the process is called gas ionization. It really is kiddie physics and there are plenty of details describing it on the internet without me wasting time.

Last edited by Orac; 03/07/16 03:16 PM.

I believe in "Evil, Bad, Ungodly fantasy science and maths", so I am undoubtedly wrong to you.
Orac #55644 03/07/16 03:16 PM
Joined: Mar 2006
Posts: 4,136
P
Megastar
Offline
Megastar
P
Joined: Mar 2006
Posts: 4,136
Originally Posted By: Bill S.
In other words; the photons are not emitted from the electron source, they result from reaction between electrons and gas. so they must curve with the electron beam.


Originally Posted By: orac
Bingo and all I did was be mean and withhold that the beam is normally invisible smile


I don't think you withheld that info , I think you just
didn't know about it.

so ... anyway now you agree that the photons do curve
with the electron beam... LOL laugh

that should have been made evident in the video that showed
the green laser being curved by the magnet.







3/4 inch of dust build up on the moon in 4.527 billion years,LOL and QM is fantasy science.
paul #55645 03/07/16 03:18 PM
Joined: May 2011
Posts: 2,819
O
Megastar
Offline
Megastar
O
Joined: May 2011
Posts: 2,819
Originally Posted By: paul
so ... anyway now you agree that the photons do curve with the electron beam... LOL laugh

that should have been evident in the video that showed
the green laser being curved by the magnet.

If that is what you believe it shows and that I agree then fine, no argument from me smile

I will accept that is what you believe just like those Dinosaurs on the Ark .. wink

Now can we get back to discussing stuff more intelligent than this as I am not remotely interested in Paul physics anymore than I am interested in Marosz physics. I may have to considering sacrificing a baby Jesus to see if we can purify the thread shortly.

Last edited by Orac; 03/07/16 03:28 PM.

I believe in "Evil, Bad, Ungodly fantasy science and maths", so I am undoubtedly wrong to you.
Orac #55646 03/07/16 04:03 PM
Joined: Mar 2006
Posts: 4,136
P
Megastar
Offline
Megastar
P
Joined: Mar 2006
Posts: 4,136
Quote:
Now can we get back to discussing stuff more intelligent than this.


not just yet , theres still a problem.

electrons are not light , only photons are light.

since the path of a photon can be curved by a magnet
as evidenced in the green laser video
then my thoughts on this is that as the electrons of the
atoms in the gas molecules become excited by photons
or electrons and then emit a photon , the emitted photons
are following the same curve that the electrons follow
resulting in a directed chain of excitation of the
electrons of atoms followed by the re-emission of photons
that follow the same electrically induced path .

this would maintain the focused beam of light that you do
see and would not result in a diffusion of light caused by undirected or unfocused light.

I think that you are thinking more about a chain of
random non directed non focused events causing the
light beam that we see.

the only basic difference between a old style residential incandesant light bulb and a electron gun is that the electron gun focuses the particles that exit the apparatus and the
old style light bulb doesn't.

focus is very important in lasers and crt tubes and electron guns .. without focus theres nothing but diffusion of particles whether they are photons or electrons.


3/4 inch of dust build up on the moon in 4.527 billion years,LOL and QM is fantasy science.
Orac #55647 03/07/16 04:18 PM
Joined: Mar 2006
Posts: 4,136
P
Megastar
Offline
Megastar
P
Joined: Mar 2006
Posts: 4,136
Quote:
I won't waste my time explaining things


definition = you don't know how things happen so you
cant explain how things happen.

Quote:
I will accept that is what you believe just like those Dinosaurs on the Ark


if you wanted to pay the shipping of a thousand dinosaurs
from new York to hong kong would you ship tiny coffee cup
sized dinosaurs or fully grown dinosaurs?

so I'm thinking that a few cubic meters of space is all that
would be needed for the dinosaurs unless there were millions
of species that lived on land.

you really need to develop thought patterns to cope with
situations that you encounter.





3/4 inch of dust build up on the moon in 4.527 billion years,LOL and QM is fantasy science.
paul #55648 03/07/16 04:44 PM
Joined: May 2011
Posts: 2,819
O
Megastar
Offline
Megastar
O
Joined: May 2011
Posts: 2,819
I think I am supposed to care but about all of the above, but all I see is focused laughs.

Why don't you take up drawing kiddie drawings like Marosz that helps apparently laugh

You are on ignore (TSTIW .. My 3rd commandment), I am not sure why you keep referencing things back to me.

Maybe the coloured electron beams were on the Ark with the Dinosaurs to keep the eggs warm laugh

Think I'm too sarcastic? Watch me pretend to care ... moving on.

Last edited by Orac; 03/07/16 05:13 PM.

I believe in "Evil, Bad, Ungodly fantasy science and maths", so I am undoubtedly wrong to you.
Orac #55649 03/07/16 10:44 PM
Joined: Mar 2006
Posts: 4,136
P
Megastar
Offline
Megastar
P
Joined: Mar 2006
Posts: 4,136
Quote:
I think I am supposed to care


I cant think of a reason why you would want to start
caring , you haven't cared up to this point and I think
that as long as you can earn money from the BS science
you will blindly follow all the other non caring money
earners riding the magic show train.

Quote:
with the Dinosaurs to keep the eggs warm laugh


there you go !!! although you didn't know it you just
experienced a thought process ... I never did mention
eggs ... you did.

cant beat that if they were available , why not because
eggs require no food until they hatch.

innovation.


3/4 inch of dust build up on the moon in 4.527 billion years,LOL and QM is fantasy science.
Bill S. #55650 03/08/16 12:16 AM
Joined: Aug 2010
Posts: 3,570
B
Bill S. Offline OP
Megastar
OP Offline
Megastar
B
Joined: Aug 2010
Posts: 3,570
Originally Posted By: Paul
the emitted photons
are following the same curve that the electrons follow


Paul, wouldn't the photons you see have to be emitted roughly at right angles to the direction of travel of the electrons, or you would not see them. The paths they would follow would be from the point of interaction to your eye, not the curve of the electron beam.


There never was nothing.
paul #55652 03/08/16 02:11 AM
Joined: May 2011
Posts: 2,819
O
Megastar
Offline
Megastar
O
Joined: May 2011
Posts: 2,819
Originally Posted By: paul
I cant think of a reason why you would want to start caring , you haven't cared up to this point and I think
that as long as you can earn money from the BS science
you will blindly follow all the other non caring money
earners riding the magic show train.

Repeat spam count 3

Bill S seems interested discuss away with him. I don't need the spam or the responses back to me as I have already told you. You are either trolling or having comprehension issues as this would be the fifth or sixth time I have had to tell you that.

Last edited by Orac; 03/08/16 03:44 AM.

I believe in "Evil, Bad, Ungodly fantasy science and maths", so I am undoubtedly wrong to you.
Bill S. #55654 03/08/16 04:13 AM
Joined: Mar 2006
Posts: 4,136
P
Megastar
Offline
Megastar
P
Joined: Mar 2006
Posts: 4,136
I think that what we see is the interaction or collision
of photons or electrons with the electrons of the gas atoms
and the light that we see is emitted by the collision
because of absorption and is visible in all directions like the light that is visible in all directions when the electrons pass through the glass of the tube while colliding with the electrons of the atoms in the glass tube.

the more I think about it the more light reminds me of a
radio wave or any other wave , why should light waves
be different from all other waves...

radio waves , sound waves , ocean waves ...
all radiate from the center in all directions

the only wave that we treat differently is light waves...

I may be wrong about that and I am tired I may read this
tomorrow and delete it ...

that would suggest that I was wrong though.

Im thinking Im right or close.


3/4 inch of dust build up on the moon in 4.527 billion years,LOL and QM is fantasy science.
paul #55656 03/08/16 12:36 PM
Joined: Aug 2010
Posts: 3,570
B
Bill S. Offline OP
Megastar
OP Offline
Megastar
B
Joined: Aug 2010
Posts: 3,570
Quote:
the only wave that we treat differently is light waves...


How are we treating light waves differently from others? What we are saying is that from their origin they radiate in all directions, but the only ones we see are those that propagate directly to our eyes.

The fact that successive emissions, in this case, come from points that lie on the curved trajectory of an electron beam does not mean that the light is being “bent”.


There never was nothing.
Bill S. #55657 03/08/16 01:00 PM
Joined: Mar 2006
Posts: 4,136
P
Megastar
Offline
Megastar
P
Joined: Mar 2006
Posts: 4,136
check

Quote:
What we are saying is that from their origin they radiate in all directions, but the only ones we see are those that propagate directly to our eyes.


so wouldn't this mean that a single interaction between a
single electron in the electron beam and a single electron of a gas molecule generates an untold number of light photons
that propagate in all directions away from the point of
interaction?

there would be trillions upon trillions of different directions that the light can be viewed from depending on
the viewing distance from the viewed light so wouldn't
there need to be a photon of light traveling in each of those
trillions upon trillions of directions?

what do you think?

BTW , there is a set precise number of electrons that
can interact with the gas molecules in the tube.

but there is no set precise number of directions that the
light that we can see can be viewed from.

so Im thinking that what we are seeing is not multiple
interactions that cause a single photon emission but
single interactions that cause multiple photon emissions.










3/4 inch of dust build up on the moon in 4.527 billion years,LOL and QM is fantasy science.
Bill S. #55659 03/08/16 03:06 PM
Joined: Aug 2010
Posts: 3,570
B
Bill S. Offline OP
Megastar
OP Offline
Megastar
B
Joined: Aug 2010
Posts: 3,570
Quote:
so wouldn't this mean that a single interaction between a single electron in the electron beam and a single electron of a gas molecule generates an untold number of light photons that propagate in all directions away from the point of interaction?


I don’t know enough about QM to say if these reactions are electron/electron interactions, or electron/atom interactions. I would suspect that the latter would be more likely.

Quote:
there would be trillions upon trillions of different directions that the light can be viewed from depending on the viewing distance from the viewed light so wouldn't there need to be a photon of light traveling in each of those trillions upon trillions of directions?


That sounds reasonable. I think one of the difficulties is that we try to look at single electrons and single photons, but the chances are that the reactions would involve trillions of both. In fact they would need to in order to be visible.

Quote:
so Im thinking that what we are seeing is not multiple interactions that cause a single photon emission but single interactions that cause multiple photon emissions.


Perhaps more likely, it would be multiple interactions, causing multiple emissions.


There never was nothing.
Bill S. #55660 03/08/16 04:31 PM
Joined: Mar 2006
Posts: 4,136
P
Megastar
Offline
Megastar
P
Joined: Mar 2006
Posts: 4,136
Quote:
I don’t know enough about QM to say if these reactions are electron/electron interactions, or electron/atom interactions. I would suspect that the latter would be more likely.


from my understanding or the way it has been explained to
me is that an electron collides with an electron of an atom
this results in either the loss of the impacted electron
causing the atom to become an ion or causes the impacted
electron to change its energy level and move to a higher
orbit around the atom.

Quote:
That sounds reasonable. I think one of the difficulties is that we try to look at single electrons and single photons, but the chances are that the reactions would involve trillions of both. In fact they would need to in order to be visible.


what I was talking about was the result of a single interaction
between a electron of an atom and a electron in the electron
stream.

so that a single interaction would result in a unknown
or infinite amount of photons of light or a better
description might be that a photon or light wave flash
occurs and the infinite amount of photons propagate
in straight lines away from the origin of the flash
in every possible direction.

Quote:
Perhaps more likely, it would be multiple interactions, causing multiple emissions.


Im assuming that you were talking about the interactions of
more than a single electron in the above.
because I don't think that a single electron could react
more than once because of absorption.

I think that the negatively charged electron would
give its charge to the impacted electron or that's the
way I understand it.

so you must have been thinking about the interactions of
more than a single electron.

I had to use the word infinite because I couldn't think of a
better word...


3/4 inch of dust build up on the moon in 4.527 billion years,LOL and QM is fantasy science.
paul #55661 03/08/16 05:30 PM
Joined: May 2011
Posts: 2,819
O
Megastar
Offline
Megastar
O
Joined: May 2011
Posts: 2,819
WOW I am impressed boys you are actually working thru this ... hmm lets give you some hints.

Not a great drawing but does this help you think more about direction of emission (Why it might align like that)


Now you have worked out there can possibly be more than one photon so exactly how many might there be ... lets give hints

1.) Conservation of energy

2.) Each photon emitted will have an energy of E=hv
h = 6.626 x 10E-34 Js (this is Planks constant)
v = frequency of light emitted ... hint light color chart

3.) The electron kinetic energy when accelerated from an electron gun is fairly simple formula.

eV = 1/2mv^2

m = 9 x 10E-31kg (The mass of an electron)
e = 1.6 x 10E-19C (The standard electron charge)
V = Voltage of the electron gun (ohhh something you can change)
v = speed of the electron accelerated by that voltage.

Nominate the gun voltage (usually like 20KV) and you can calculate the electron velocity and hence it's Kinetic energy it has when it slams into the gas atom.

KE = 1/mv^2

Final hint: Look at energy of point 3 and ask how many of point 2 can it make given point 1 .. the answer isn't infinite smile

Prediction: If you got it all sorted there is an obvious prediction what will happen if you vary the electron gun voltage and you can test it smile

Last edited by Orac; 03/08/16 05:46 PM.

I believe in "Evil, Bad, Ungodly fantasy science and maths", so I am undoubtedly wrong to you.
Orac #55663 03/08/16 09:03 PM
Joined: Mar 2006
Posts: 4,136
P
Megastar
Offline
Megastar
P
Joined: Mar 2006
Posts: 4,136
I suppose the math must be wrong then orac.

think of a transparent sphere with a radius of the furthest star that we can see.

now think about being on any point on the sphere
looking in at its center.

if the electron gun is located at the center of the sphere
and is switched on just long enough for a single flash to
occur.

and for now lets say that the radius of the sphere is 1 light year.

in 1 year the photons of light would reach you.
and you could see them from any point on the surface of
the sphere if you had a powerful enough telescope.

now how many photons are reaching the transparent sphere?

does the math tell you that the photons would not be
observable from any point on the sphere?

if so then how small would a sphere need to be in order
to observe the photons from any point on the sphere?

the sphere just gave me an idea , perhaps light is
like a infinitely expanding sphere and the thickness
of the light sphere is what we currently think of as
the length of a light wave.

when you think of a water wave as in
a tiny drop of water that is dropped onto a smooth water surface the result is waves that increase in radius and these waves are uniformly shaped and they travel outward in all horizontal directions on the waters surface from the center of where the water drop was dropped.










3/4 inch of dust build up on the moon in 4.527 billion years,LOL and QM is fantasy science.
paul #55665 03/09/16 03:30 AM
Joined: May 2011
Posts: 2,819
O
Megastar
Offline
Megastar
O
Joined: May 2011
Posts: 2,819
Now you have run across the conundrum smile

You are correct in your post but it was my best effort to make some sense classically of it. It was a shame you didn't do the bit about the number of photons which would have shown you another problem.

The problem was actually first considered with stars being so far away. If you put a star a long way away like 13.2 billion light years then if photons are like bullets at that distance you would only occasionally see the star on the rare time a photon comes in your specific direction. Even with the shear number of photons being emitted from a sun at that distance it would be true.

The question is usually innocently asked like this "As light from a star spreads out, do gaps form between the photons?"

The alternative is to turn light it into a wave but then you have problems with particle like behaviour mirrors, optical lense and lasers etc.

You have four choices
i) Light is a Particle-only
ii) Light is a Wave-only
iii) Light is a Both-particle-and-wave
iv) Light is Neither-wave-nor-particle

You may care to first search for "evidence light is a particle" and "evidence light is a wave" which will probably list relevant facts and experiments.

I actually can't help you with this problem and not because I don't want to but I don't know a solution in your physics. So you will need to work through the examples and experiments from your searches and see if your choice of solution works.

Three situations you may like to consider

i) How a mirror works especially with a wave only version rather than a photon (ray) version. Scientists do lots of lies and hand waving on this so look carefully at any answer (esp drawings) and it may pay to look at wave behaviour in a ripple tank.

ii) Two people standing either side of a photon emission can both people see it? Think about energy as the photon is absorbed and there is often lots of hand waving about this.

iii) A radio wave can be blocked by flywire screen mesh but an x-ray will require a fair amount of solid lead .... why the difference?

Finally I will make a statement which may seem to be selective but you need to look beyond that, I am not favouring any answer.

There is nothing special about light, radio waves exhibit the exact same behaviours they are just no readily encountered by layman. There are situations it is easier to consider the radio wave a particle but the particle is meters in diameter which is very weird for us.

Ultimately you will come up with and answer from those 4, so let me know.

In my physics take any of those answers you like, strangely it doesn't matter laugh

There is a hint in that to concentrate on your argument rather than what poor old Orac thinks, I have no position on this nor do I care about the answer.

Last edited by Orac; 03/09/16 06:24 AM.

I believe in "Evil, Bad, Ungodly fantasy science and maths", so I am undoubtedly wrong to you.
Orac #55674 03/09/16 07:27 PM
Joined: Mar 2006
Posts: 4,136
P
Megastar
Offline
Megastar
P
Joined: Mar 2006
Posts: 4,136
hmmm

(v) light is a infinitely expanding sphere.

(a) there are different intensities / frequencies of light
and the thickness of the sphere reflects the intensity
or frequency of the light.

(b) a sphere of light expands in all directions relative to the
point of its origin.

(c) the speed of the expansion of light has no limits nor does
the size of the expanding sphere of light.

its easy as

(1)


(2)


(3)

do re me

thats how easy light can be


sick



3/4 inch of dust build up on the moon in 4.527 billion years,LOL and QM is fantasy science.
Joined: May 2011
Posts: 2,819
O
Megastar
Offline
Megastar
O
Joined: May 2011
Posts: 2,819
YOUR ANSWER ABOVE DISAGREES WITH YOUR PREVIOUS ANSWER "THE LIGHT WAS FOLLOWING THE BEAM AND BEING DEFLECTED".

So have you gone to individual atoms emitting spherical emission as your answer?

Extension question is light a wave or a particle, both or none?

That is after all the question, and your answer is a little light on detail smile

So how does our colored electron beam work and then how do things like laser cooling work. Your answer for laser cooling ages ago had photon particles hitting into the atoms.

You are the only one who can answer these, and you have been trying to drag me in to this classical physics stupidity for about the last 20 posts .... SO NOW DISCUSS IT.

Now suddenly you have gone all quite just when it got interesting ... or are you a bit off color laugh

Paul, you seem to have a problem when you don't have me to argue against smile

If you are going on I suggest I give you the simple kiddie experiment we use, which gives you no place to hide. You are left with only a couple of options and they all have consequences. Let me know if you want it.

So are we done with this classical physics trash now? Can we move along to something more intelligent. As I said the whole discussion was waste of time and effort, and we end now at "do re me" because that is all we could ever end at.

Last edited by Orac; 03/10/16 01:58 AM.

I believe in "Evil, Bad, Ungodly fantasy science and maths", so I am undoubtedly wrong to you.
Joined: Mar 2006
Posts: 4,136
P
Megastar
Offline
Megastar
P
Joined: Mar 2006
Posts: 4,136
Quote:
YOUR ANSWER ABOVE DISAGREES WITH YOUR PREVIOUS ANSWER "THE LIGHT WAS FOLLOWING THE BEAM AND BEING DEFLECTED".


no it doesnt dissagree.

the sphere of light can be deformed by gravitational
or magnetic influence as evidenced in the green laser video.

a focused beam of light is a oblong sphere of light that
has been deformed due to the influence of gravitational
or magnetic influence.

so the emitted light spheres are deformed in the electron gun
due to magnetic influence the same way that the path of the electrons are deformed by magnetic influence.

the shell of the light sphere interacts with the gravitational
and magnetic influences that it encounters in its environment.

I cant seem to think of a proper example to use to describe
the motion and acceleration of the environmentally influenced shell of the light sphere.

think of the thin shell of a extremely elastic rubber balloon.

the balloon is filled with a slightly compressed gas and
the gas supports the shell of the balloon so that the shell
forms a perfect sphere.

the gas molecules inside the balloon interact with a magnetic field.

when the argon gas in the electron gun emit a sphere of
light due to the interaction with a electron in the electron stream the gas molecules in the balloon are accelerated
by the magnetic field in the electron gun and the individual
gas molecules inside the balloon are accelerated according to the intensity of the field resulting in a oblong sphere of curved light.

Quote:
So have you gone to individual atoms emitting spherical emission as your answer?


the only way that light can be emitted is if a atom emits the
light , and light is a sphere.

Quote:
Extension question is light a wave or a particle, both or none?


the wave that is used to describe light is most
likely simply a means of description that describes a single
flash of light emission from the moment of the flash of light
emission up to the maximun intensity of the flash and down to the moment that the flash has passed by.

the above description includes the lenght and intensity of a single flash of emitted light as the shell of the sphere
of light passes a single point within a time range.

a particle of light to me says a perfectly straight line
and since I am hardheaded and strickly classic for now
I'm going to go with particle until I find a better word.

Quote:
Now suddenly you have gone all quite just when it got interesting ... or are you a bit off colo


unlike yourself orac I am a human being that requires
sleep , so try to keep you panties from getting so
wadded up in knots when I leave for a few hours to sleep.

you should have your programmers insert a timer function
to turn your program off for a few hours a day to avoid the
lonelyness that you seem to be experiencing.

cry




3/4 inch of dust build up on the moon in 4.527 billion years,LOL and QM is fantasy science.
Joined: May 2011
Posts: 2,819
O
Megastar
Offline
Megastar
O
Joined: May 2011
Posts: 2,819
So a laser is just a really really really stretched ellipse so it just looks like a line??????? I am trying to understand you here.

Originally Posted By: Paul
so try to keep you panties from getting so
wadded up in knots when I leave for a few hours to sleep

I really don't care about any of this I already know the problems no matter what logic you use we will come unstuck. If we don't you just solved the biggest problem in classical physics and what ultimately is wrong with it. I win anyhow as I will write it up, credit you and we collect the Nobel prize and we split the money and glory smile

The point you keep failing to understand is I am open to any answer so long as it makes sense, and you can convert me and I am on your side wink

You need to explain another thing to me does the power of your sphere of light work?

Let me describe a setup:

We have an atom that produces a single photon flash (its called a single photon source) and you are saying the flash spreads out as a sphere like a radio wave, I think. Now in a circle several meters away I have 20 photovoltaic cells (solar cells) around in an even ring each the same distance from the photon flash facing directly at it.

So an icosagon each of the 20 points on the circle has a solar cell facing the single photon source in the middle


Now do each of the 20 cells register a voltage 1/20th or less of the value compared to if I had the source right close to the single photon source?

The voltage of a solar cell is directly proportional to the power of the light hitting the cell and it is instant so I can understand how your power of the sphere emission works.

As you can probably guess I already know what happens here (this is a classic university experiment), so be careful get it wrong and your theory is dead smile

If you want to do this experiment at home (to check the universities) I can give you the radio wave version which behaves exactly the same as what we do here with light. It drives the kids nuts when they are shown the experiment because it only makes sense if you adapt things.

Last edited by Orac; 03/10/16 04:16 PM.

I believe in "Evil, Bad, Ungodly fantasy science and maths", so I am undoubtedly wrong to you.
Joined: Mar 2006
Posts: 4,136
P
Megastar
Offline
Megastar
P
Joined: Mar 2006
Posts: 4,136
Quote:
So a laser is just a really really really stretched ellipse so it just looks like a line??????? I am trying to understand you here.


no no no , I was talking about the way a sphere of light
would behave inside a electron gun.

a laser would only allow light to escape out through the
light emission end of the laser.

and laser light is simply focused intensified light
so a light sphere would behave the same way as a sphere of light would outside of the laser.

Quote:
Now do each of the 20 cells register a voltage 1/20th or less of the value compared to if I had the source right close to the single photon source?


isnt it 6kW / day 1300 watts / hour ( 5 hour day ) per
square meter solar irradiance ... from the sun?
the strongest solar pannels today are apx 150 - 200 (watts)
per apx square meter.

the efficiency of a photoelectric cell is apx 1/7th of the
possible energy from the sun.

but they are getting better.

so if the light in the center used 8 kW / day and the achieved
light irradiance was 6kW / day per square meter and each of
the 20 photocells were 1/20th of a square meter then the photocells might register a total of 175 watts / hour or 8.75 watts / hour each.

Quote:
The voltage of a solar cell is directly proportional to the power of the light hitting the cell and it is instant so I can understand how your power of the sphere emission works.


solar cells are no where near that efficient , if they were
I would fill the inside of a clothes closet with a few
solar pannels and put a 500 watt lamp in it and power
the rest of the house with it...

BTW , you can keep the nobel prize but I will be wanting the
money.




3/4 inch of dust build up on the moon in 4.527 billion years,LOL and QM is fantasy science.
Joined: May 2011
Posts: 2,819
O
Megastar
Offline
Megastar
O
Joined: May 2011
Posts: 2,819
You changed the experiment so lets add more detail to bring it to what we are after.

i) The room in which our setup is in pitch black.
ii) The cells are producing zero volts obviously there is no light at all.
iii) Each cell is connected to a storage oscilloscope because we are going to have to measure fast.
iv) Our photon source in the middle turns on for one pulse every 10 minutes. This is like you you flicking the dimmest light globe on and off once really really fast in 10 minutes.

Our oscilloscopes will record the response of the solar cells to that one photon every 10 minutes.

The power equations in your post above you refer to are MACRO laws for continuous power situations. Like all such laws you can bend and violate them if you understand what they are really measuring which is what we are trying to do here. Those laws are correct by the way for the macro situation with normal setups but you can bend them with special setups.

We are interested in the behaviour of one photon (the smallest flick on/off) of a light pulse we can make.

So given our complete dark setup what will the oscilloscopes of each cell see in response to the single photon emission every 10 minutes.

Whats your prediction? Our money awaits if you can get it right.

To help I will tell you what doesn't happen which is the logical expectation based on the MACRO laws. Naively we (perhaps it's just me when I first saw this) would expect all the 20 cells to record a fleeting fast voltage spike settling back down to zero as the room goes black again. You might also expect if you took the panels closer to the source you would get higher fleeting voltage spike recorded by the oscilloscopes on each of the 20 cells as you would expect the power to be higher.

HINT THAT IS NOT WHAT HAPPENS ... our macro laws don't work in this situation. As I said there is a fun version you can do at home which shows the same thing which I will discuss in a minute.

The thing the experiment throws up is what is wrong with the macro laws and what are they based on. I am strictly avoiding giving you any answers just giving you the results for you to work with the raw experiment results with no interpretation. I really want my money if you can solve it.

Last edited by Orac; 03/11/16 08:43 AM.

I believe in "Evil, Bad, Ungodly fantasy science and maths", so I am undoubtedly wrong to you.
Joined: Mar 2006
Posts: 4,136
P
Megastar
Offline
Megastar
P
Joined: Mar 2006
Posts: 4,136
the experiment will work the same way as a home style
solar pannel will.

the larger collection area the larger the amount of
light energy the collector can collect and process into
electricity.

you didnt say what the size of the collection area
of the 20 photo cells was.

and you didnt say what the efficiency of the photo cells
were.

you didnt say what the emission energy of the photon was.

you didnt say what the distance from the light source
to the photo cells was.

you didnt say that the experiment would be performed
in a medium free environment.

without knowing all of the above I can only say that
light would propagate to the photo cells.

assuming that the photo cells used can pick up
the single flash of light and trigger some electrical
circuit.

I can also say that if you had 20 light activated
cameras they each should record the same amount of brightness
of the photon flash if they were all a set distance from the
photon flash.

or a thousand / million / billion miniture cameras mounted to form a sphere around the flash.

if there is a flash then they should all record the same brightness visible from the set distance to the light source.

so knowing that then if you know the amount of energy that
one of the single photo cells on the cameras produced to trigger the electrical circuit then all you need to do is to multiply the amount of energy collected and processed by the
photo cell x the number of cameras you used.

shocked









3/4 inch of dust build up on the moon in 4.527 billion years,LOL and QM is fantasy science.
Joined: May 2011
Posts: 2,819
O
Megastar
Offline
Megastar
O
Joined: May 2011
Posts: 2,819
And yet that isn't what happens smile

The cell sensitivity is set to the level of the one flash that is about all you need to know. Each panel just has the ability to "just" register the flash. The rest of your stuff is window dressing and detail we need not worry about.

Yet when the flash happens exactly one will see the flash and only one not all of them.

It's funny weird hey .... if you have one panel that is more sensitive it will see the flash more likely than the others. If you move one panel slightly closer it will see the flash with increased likelyhood.

The best way I can explain it in layman terms is in some ways it is like lightning. All the cells have the same potential to see the flash but the moment one reaches a point that it does see the flash all the energy seems to go to that cell and there is only enough energy to register on one cell.

The result here is known and done by thousands of students every year and that is what happens. It shows the breakdown of the macro laws and your answer above is the one we all initially think should be the case and indeed I thought the same should occur.

Would you like the home experiment with RF so you can do it and see the behaviour on RF?

Last edited by Orac; 03/11/16 01:26 PM.

I believe in "Evil, Bad, Ungodly fantasy science and maths", so I am undoubtedly wrong to you.
Joined: Mar 2006
Posts: 4,136
P
Megastar
Offline
Megastar
P
Joined: Mar 2006
Posts: 4,136
Quote:
The cell sensitivity is set to the level of the one flash that is about all you need to know. Each panel just has the ability to "just" register the flash. The rest of your stuff is window dressing and detail we need not worry about.

Yet when the flash happens exactly one will see the flash and only one not all of them.


LOL , very convienient! wink

a thousand pictures say a single word.

a billion pictures say the same word only louder.



3/4 inch of dust build up on the moon in 4.527 billion years,LOL and QM is fantasy science.
Joined: Mar 2006
Posts: 4,136
P
Megastar
Offline
Megastar
P
Joined: Mar 2006
Posts: 4,136
Quote:
The best way I can explain it in layman terms is in some ways it is like lightning. All the cells have the same potential to see the flash but the moment one reaches a point that it does see the flash all the energy seems to go to that cell and there is only enough energy to register on one cell.


you dont actually believe that do you?

what your saying is that all the light vanishes as soon
as the one cell detects the light... seriously?

so now we have a light sink that sucks all the light
energy into a single photo cell... its amazing to me
how advanced experiment rigging has become.

tell me , what was the purpose of the experiment?

and so that I can experience the wonders of this
certain impossible phenomenon would you mind telling
me the common name of this experiment?





3/4 inch of dust build up on the moon in 4.527 billion years,LOL and QM is fantasy science.
Joined: May 2011
Posts: 2,819
O
Megastar
Offline
Megastar
O
Joined: May 2011
Posts: 2,819
Well try it yourself at home all you need is a phone and some flashing led phone tips from your local phone store or any device that harvest RF power from the signal.

http://www.neatstuffusa.com/cellular11/Flashing_Antex.html they are a few bucks from any phone store accessory store.

Stand the phone up and move one of the tips in just close enough so it glows. Try each tip individually and make sure they glow at this point.

Now at this point if you draw a circle around the phone the power at that distance should be the same at ever point on the circle and test each tip works at each point on the circle.

Now the problem put another couple of tips on the circle anywhere you like you will get a surprise they wont all light up together.

The phone simply can't power all the tips and only some will stay on. Try any one individually and they will power but it simply can't lift too many because although the RF emission spreads out spherically the moment you start drawing power something happens and our spherical emission collapses.

The purpose of the experiment is simply to show you the behaviour which you need to explain. If we measure the field it does indeed spread spherically and you have the possibility to draw power at that point. The problem becomes when you actually start to absorb the transmission power.

Normally the transmission power is far greater than the power we draw but here we are investigating what happens if there isn't enough power.

I am not trying to convert you or convince you but simply show you what happens. It's a problem nobody has been able to solve using classical physics because your laws change, collapse or vary or something.

This has nothing to do with belief it is what happens in easy to demonstrate experiments and we know the issue it is the power.

That is why when this collapse happens it's easier for science to consider that the light was a particle that flew directly to the detector and that is why classical science has the particle behaviour of light. I won't defend it but that it's what historically they did to explain the behaviour.

Originally Posted By: paul
what your saying is that all the light vanishes as soonas the one cell detects the light... seriously?

Ok I would say the emission pattern changes I wouldn't say the light disappears. Lightning does somewhat the same thing if I had to compare it to something, it starts a sheet of voltage but once it breaks and produces actual energy it goes in a totally emission characteristic. Why does lightning hit a particular tree or building and not another.

Repeating these are experiments done on multiple forms of EM waves they all behave like it and not something that I have a choice in believing ... it happens repeatedly in all experiments.

Last edited by Orac; 03/11/16 02:16 PM.

I believe in "Evil, Bad, Ungodly fantasy science and maths", so I am undoubtedly wrong to you.
Joined: Mar 2006
Posts: 4,136
P
Megastar
Offline
Megastar
P
Joined: Mar 2006
Posts: 4,136
perhaps the RF harvester is causing the RF sphere to
collapse because of the consumption of the RF at the
location of the RF harvester and this causes the remaining
RF sphere to undergo a frequency change.

have they measured the remaining sphere to see if it has undergone a change in frequency?

Im assuming that the RF harvesters only harvest a certain
RF.






3/4 inch of dust build up on the moon in 4.527 billion years,LOL and QM is fantasy science.
Joined: May 2011
Posts: 2,819
O
Megastar
Offline
Megastar
O
Joined: May 2011
Posts: 2,819
Originally Posted By: paul
perhaps the RF harvester is causing the RF sphere to
collapse because of the consumption of the RF at the
location of the RF harvester and this causes the remaining
RF sphere to undergo a frequency change.

have they measured the remaining field to see if it has undergone a change in frequency?

The frequency is unchanged but the field collapses in the RF case it looks like the RF harvester is directly connected to the antenna by a direction link or antenna even though the antenna is unchanged.

Now obviously something similar must happen to light which is nothing more than an EM wave itself.

Originally Posted By: paul
Im assuming that the RF harvesters only harvest a certain RF.

The RF harvesters are very non frequency sensitive they are coils and a few diodes in the form I gave you. Google phone detectors they are basically homebrews of the same thing.

Measuring the result is easy coming up with a theory of why it happens is a little harder.

You can also know when the collapse will happen just follow the conservation of energy. The moment you try and draw more power than what you have available it collapses.

None of this has anything to do with QM by the way this is all very classical and there is no answer to it under QM we can simply describe the effects easier. So I can put maths around it but not tell you why it happens it just does.

Last edited by Orac; 03/11/16 02:27 PM.

I believe in "Evil, Bad, Ungodly fantasy science and maths", so I am undoubtedly wrong to you.
Joined: Mar 2006
Posts: 4,136
P
Megastar
Offline
Megastar
P
Joined: Mar 2006
Posts: 4,136
that is weird !!!

so distance matters , how about if they stagger the
radius distance of each harvester from the phone?


3/4 inch of dust build up on the moon in 4.527 billion years,LOL and QM is fantasy science.
Joined: May 2011
Posts: 2,819
O
Megastar
Offline
Megastar
O
Joined: May 2011
Posts: 2,819
Originally Posted By: paul
that is weird !!!

so distance matters , how about if they stagger the
distance of each harvester?

Yeah its weird that is why it drives people nuts.

Staggering and moving things around does change things different ones will light up etc but there is only the obvious patterns. The more sensitive ones will tend to light and the closer ones will tend to light.

In the case I gave you with the cells and the single photon we control the hell out of everything and the cell is actually a single photon receiver and still the result is somewhat random.

You were right with the spherical emission that does happen and the macro laws do work at a macro level.

The upshot here is the human eye absorbs the power of emissions so when you get down to really low levels like we are talking no two people or detectors can see the same photon there just isn't the power. The human eye BTW needs to absorb 6 to 8 photons to register light we even measured that by somewhat dubious experiments ... not going there.

I should also say that using QM you can claim you can observe a photon without destroying it but it is a bit of trickery and doesn't help with the problem outlined above.
http://www.nature.com/news/photons-detected-without-being-destroyed-1.14179

Last edited by Orac; 03/11/16 02:42 PM.

I believe in "Evil, Bad, Ungodly fantasy science and maths", so I am undoubtedly wrong to you.
Joined: Mar 2006
Posts: 4,136
P
Megastar
Offline
Megastar
P
Joined: Mar 2006
Posts: 4,136
Quote:
The RF harvesters are very non frequency sensitive they are coils and a few diodes in the form I gave you. Google phone detectors they are basically homebrews of the same thing.

Measuring the result is easy coming up with a theory of why it happens is a little harder.

You can also know when the collapse will happen just follow the conservation of energy. The moment you try and draw more power than what you have available it collapses.


ok , so this happens with RF.

does that also mean that this would happen with light?

ie...

if there were a gazillion earths set in a sphere around our
sun would this mean that only a few of the earths would
absorb the light from our sun?


3/4 inch of dust build up on the moon in 4.527 billion years,LOL and QM is fantasy science.
Joined: May 2011
Posts: 2,819
O
Megastar
Offline
Megastar
O
Joined: May 2011
Posts: 2,819
Originally Posted By: paul

ok , so this happens with RF.

does that also mean that this would happen with light?

ie...

if there were a gazillion earths set in a sphere around our
sun would this mean that only a few of the earths would
absorb the light from our sun?

Very well done .. you are one of the few I have ever seen get that straight up.

Yes the prediction would be if you stuck enough solar panels or things slightly closer and more sensitive than us to the sun then it could dim it. The number is massively impossible but the thought is it might be more interesting to try on a light bulb and that process is underway. So the question is can you dim a light bulb by simply having lots of observers each absorbing photons. The observers being cells we can also easily turn on and off by covering them and the dimming would easily be measurable.

Whether it works like that or not is unknown because we are talking scale here. Lightning bolts only grow to a certain size and width they are constrained by something. Whether this effect holds in the macro scale is a completely other story.

I can see you get the problems it opens up and you will see reports on this stuff over the coming years. We never thought to try these large scale things until we worked out what was happening with the micro scale stuff. For my part I am dubious it works like this on large scale I think there is some sort of limit like with lightning and these are micro effects.

Some like Tesla clearly believed the effects could be made macro and I think we now understand what he was trying to do even though we know his theory was slightly wrong. Myself I need proof of macro scale effects of this stuff.

Last edited by Orac; 03/11/16 03:10 PM.

I believe in "Evil, Bad, Ungodly fantasy science and maths", so I am undoubtedly wrong to you.
Joined: Mar 2006
Posts: 4,136
P
Megastar
Offline
Megastar
P
Joined: Mar 2006
Posts: 4,136
I think the thickness of a lightning bolt is constrained
by the vacuum it causes in the atmosphere as it vaporizes the air particles once the connection has been made from ground
to cloud or visa versa.

so perhaps there is something in the charge sea that lends
itself to be the connection between the phone and the RF harvester that completes the circuit and collapses the sphere.

ions?


3/4 inch of dust build up on the moon in 4.527 billion years,LOL and QM is fantasy science.
Joined: May 2011
Posts: 2,819
O
Megastar
Offline
Megastar
O
Joined: May 2011
Posts: 2,819
Originally Posted By: paul
so perhaps there is something in the charge sea that lends itself to be the connection between the phone and the RF harvester that completes the circuit and collapses the sphere.

ions?

Yeah we simply don't know you are clearly thinking along the same lines as science that there should be some measurable detail.

It has frustrated us since 1980 when we first confirmed the photon behaviour.

Last edited by Orac; 03/11/16 03:16 PM.

I believe in "Evil, Bad, Ungodly fantasy science and maths", so I am undoubtedly wrong to you.
Joined: Mar 2006
Posts: 4,136
P
Megastar
Offline
Megastar
P
Joined: Mar 2006
Posts: 4,136
Quote:
the thought is it might be more interesting to try on a light bulb



heres a group experiment that could be performed in a
stadium.

get everyone to look at a single light bulb glowing
in the center of the football field and see if the light bulb
goes out... LOL

now get them to blink in unison ...

what do you think?


and take a survey of how many people could see
the light , there should be a set number of people who
could see it.

given that our eyes should absorb a given amount.

if you have a light source
located at the center of a small sphere the entire
inside surface of the sphere should absorb the light
and heat up if it were painted black.

and reflect the light and not heat up because of the
light if it were painted white.

Im thinking that the color of the light of the harvester
might be interfering with the RF transmitted from the phone
have there been any test to see if it is the light on the
harvester that is causing the other harvesters to not
light up ...

wire the lights away from the circle or block the light
so that the harvester light can not interfere with the RF
signal.

just pickin a straw.

Im going to think about all of this for a few hours and
hopefully it wont drive me crazier than I already am.

meanwhile Im going to have lunch.

Im thinking there may be a way to set up the light
experiment to get all of the photo cells to register.

so Im going to think of a way to do it.



3/4 inch of dust build up on the moon in 4.527 billion years,LOL and QM is fantasy science.
Joined: May 2011
Posts: 2,819
O
Megastar
Offline
Megastar
O
Joined: May 2011
Posts: 2,819
Yeah we have jokes like that as well smile

The numbers in reality are 100K people in a stadium only absorb 600K-800K photons per second and 8W light produces 2.4E19 photons per second .. just so you can see the scale and why it would not work even if the micro prediction held.

My favourite is see a watch pot really doesn't boil because you steal all the energy laugh

I like you think the effects probably don't survive to macro scale but it would be nice to know what controls the cross over exactly.

Last edited by Orac; 03/11/16 03:44 PM.

I believe in "Evil, Bad, Ungodly fantasy science and maths", so I am undoubtedly wrong to you.
Joined: May 2011
Posts: 2,819
O
Megastar
Offline
Megastar
O
Joined: May 2011
Posts: 2,819
Originally Posted By: paul
Im thinking that the color of the light of the harvester might be interfering with the RF transmitted from the phone have there been any test to see if it is the light on the harvester that is causing the other harvesters to not
light up ...

wire the lights away from the circle or block the light
so that the harvester light can not interfere with the RF
signal.

just pickin a straw.

In the proper lab we don't use the home sort of harvester we use atoms to do the absorbing. The usual is pentacene (https://en.wikipedia.org/wiki/Pentacene) which has a nice exciton emission you can easily detect.

So we convert the emission directly to an electrical signal.

We do eliminate that sort of thing and it still behaves the same.

Oh now you understand what was happening you will probably be able to understand what a maser is and why it works like a laser.

Originally Posted By: https://en.wikipedia.org/wiki/Maser
Contemporary masers can be designed to generate electromagnetic waves at not only microwave frequencies but also radio and infrared frequencies

You will also get why and how it works across all those frequencies now and confirmation the effect is real at a micro level. Basically you control which atoms are likely to see an emission and by doing so build a population inversion for the stimulated emission which you output for use. There are some designs that don't work like this they use different mechanisms but it is the most common way to do it.

Last edited by Orac; 03/11/16 04:04 PM.

I believe in "Evil, Bad, Ungodly fantasy science and maths", so I am undoubtedly wrong to you.
Joined: Mar 2006
Posts: 4,136
P
Megastar
Offline
Megastar
P
Joined: Mar 2006
Posts: 4,136
so I have a headache now , thanks orac. frown

my brain has told me that the RF that is being consumed
by the RF harvester can be compared to a air valve opening
on a compressed air tank that has 20 air pressure guages
and 20 pressure release valves attached in a circle around it.

the pressure release valves will only open at a certain
pressure and after the first pressure valve has opened then
the remaining 19 valves will not open because the air
pressure drops inside the tank , so an individual pressure valve will never open unless it is the first one to open.

from a engineers viewpoint that issue can be solved by
ensuring that all 20 of the valves are opened at the same time.

the purpose of doing this would be to equally distribute
the escaping pressurized air for stabilization or to reduce
material stress.

now , the pressure valves can be fitted with orifices
and pressure regulators that only allow a certain amount
of pressurized air to pass through the orifice in a given
time no matter how large the pressure is in the tank.

now applying the above thoughts I get the below
but using light vs RF and the below may work with RF
Im not sure yet.

we now know that we can control the speed of light using
a medium so there is the pressure regulator valve and
the pressure release valve and the pressure orifice.

by slowing down the light using 20 mediums that the light
passes through and then collecting the light and converting
the light you remove the ability of a single photo cell
to absorb all of the light energy.

the light will be trapped inside the medium until it
exits the medium but it doesnt need to exit the medium
to be detected.

also the exit time of light exiting the 20 mediums can be staggered over a time period using different thicknesses of
mediums that would allow Time for each individual photo
cell to collect and convert the light into a capacitance array
to be used to compare the emission energy of the light source and the energy collected in the capacitance array.


what do you think?



3/4 inch of dust build up on the moon in 4.527 billion years,LOL and QM is fantasy science.
Joined: May 2011
Posts: 2,819
O
Megastar
Offline
Megastar
O
Joined: May 2011
Posts: 2,819
It gave me a headache trying to understand you as well smile

I followed all the top bit it is very engineering but it got harder to follow you the further down I went.

Originally Posted By: Paul
the light will be trapped inside the medium until it
exits the medium but it doesnt need to exit the medium to be detected.

I don't understand the point made in this quote above. You can detect the light in and out of the medium, does that mean light has two forms that have subtle differences?

Originally Posted By: Paul
also the exit time of light exiting the 20 mediums can be staggered over a time period using different thicknesses of mediums that would allow Time for each individual photo cell to collect and convert the light into a capacitance array to be used to compare the emission energy of the light source and the energy collected in the capacitance array.

This wont work, there are some things that happen with light when it enters a medium and it doesn't quite behave the same.

However I think I can show you what you are trying to do which is called an optical Maxwells demon

http://phys.org/news/2016-02-physicists-photonic-maxwell-demon.html

They use observation of a photon to indeed charge a capacitor.

The idea goes way back to 1867 and has been a theoretical idea it just took a long time for anyone to work out how to construct an experiment to do it. The experiment has application in testing some QM concepts and ideas to do with thermodynamics which groups are trying it at the moment.

If I understand you that is what you were trying to do, slow light down to be able to create a measured charge separation. The group above just did it a very different way.

The absurdity of that experiment still does my head in. You absorb energy by observation creating a power difference. You then use the power difference to charge a capacitor, which you can then use to power other things ... WTF.

Consider the joke if you take that to the largest macro scale, us humans observing gives energy to the universe to use elsewhere. Oh got a good joke, stop looking at things Paul you are making the universe expand.

All they need now is to tell me it can be done at 100% efficiency and I slash my wrists and give up on the universe smile

Last edited by Orac; 03/12/16 03:12 AM.

I believe in "Evil, Bad, Ungodly fantasy science and maths", so I am undoubtedly wrong to you.
Joined: Mar 2006
Posts: 4,136
P
Megastar
Offline
Megastar
P
Joined: Mar 2006
Posts: 4,136
Quote:
the light will be trapped inside the medium until it
exits the medium but it doesnt need to exit the medium to be detected.


Quote:
I don't understand the point made in this quote above. You can detect the light in and out of the medium, does that mean light has two forms that have subtle differences?


I was just saying that the light doesnt need to be
detected while it is trapped inside the medium.

the speed of light slows through a medium.
so the thickness of the medium causes the travel time
of light inside the medium to increase.

because the speed of light slows apx 40% through glass.

so before light enters a glass medium the light is
traveling at its normal speed through its current
medium such as air.

lets say we want to allow light to travel a distance
of 3 cm from point A to point B

and located between point A and point B there is a
1 cm thick glass medium that the light must pass through
in order to reach point B.

the light moves away from point A towards point B

and lets say the light has a speed of 300000 km/s
before the light enters the glass medium.

the light then enters the 1 cm thick glass medium.

the light then slows by 40% to 180000 km/s as it enters
the glass medium.

so while the light is traveling through the glass
medium the speed of the light is 180000 km/s vs 300000 km/s.

obviously when the light slows through the glass medium
then the travel time of the light from point A to point B
would be longer than the travel time of the light from
point A to point B if the 1 cm glass medium were not there.

so the thickness of the glass medium would determine the
amount of Time delay from point A to point B.

the light then exits the 1 cm glass medium and it
resumes its normal speed of 300000 km/s.

if a photo cell is located at point B then there
would be a Time delay of the light reaching the photo
cell.

and if there were 20 photo cells in a circle
like in the experiment you posted about where all
the energy from the light was directed into a single photo
cell.

then this would be a way to store the light inside the
glass medium long enough to allow each of the 20 photo cells
to collect and convert the light energy if each of the
photo cells had a glass medium between point A and point B.

ie...

1 light source. (point A )
20 glass mediums. ( of the same or different thicknesses )
20 photo cells. (point B )
1 capacitance array to collect and store the electricity.

this could be done 1 at a time or all 20 at the same time.

because the thickness of the glass medium would determine
when the light would reach the photo cells.

so if you have 20 different thicknesses of glass then
you would have 20 different time periods that the light
would reach the photo cells.

concerning the experiment you posted about where all the energy was directed into the single photo cell and no one
could understand why this happened the above does not
explain why this happens but it would provide a work around
to avoid it from happening.

and perhaps even more !!! cool



3/4 inch of dust build up on the moon in 4.527 billion years,LOL and QM is fantasy science.
Joined: May 2011
Posts: 2,819
O
Megastar
Offline
Megastar
O
Joined: May 2011
Posts: 2,819
Originally Posted By: paul
so if you have 20 different thicknesses of glass then you would have 20 different time periods that the light
would reach the photo cells.

Gotcha now and no it won't and doesn't work.

IBM did that trick on a silicon chip in 2005
(https://www-03.ibm.com/press/us/en/pressrelease/7956.wss)

It slowed light down to 1/300th of its usual speed and they then set a cell at it's end and one in which the cell could see the light in the normal way and time.

What happened was that the normal one was just more likely to see the photon the one with the massive delay saw the photon less times. However if the delayed one was the only detector (you covered the normal one) the delayed photon was always seen.

It is was that weird behaviour again and the time delay just reduces the chance it will see the photon.

I personally have also done this experiment on RF using radio frequency delay transmission lines and the same thing happens if you harvest the RF.

Originally Posted By: paul
concerning the experiment you posted about where all the energy was directed into the single photo cell and no one could understand why this happened the above does not
explain why this happens but it would provide a work around
to avoid it from happening.

Yes we expected that as well but it doesn't work.

Last edited by Orac; 03/13/16 03:14 PM.

I believe in "Evil, Bad, Ungodly fantasy science and maths", so I am undoubtedly wrong to you.
Joined: Aug 2010
Posts: 3,570
B
Bill S. Offline OP
Megastar
OP Offline
Megastar
B
Joined: Aug 2010
Posts: 3,570
I have neither the time, nor the inclination, to wade through this thread to find it, but I think the question was raised as to whether, in the photon model of light, gaps would arise between photons when light travelled over a very long distance.

Christopher Baird deals with this, here:

http://sciencequestionswithsurprisingans...en-the-photons/

I would appreciate comments on this.


There never was nothing.
Joined: Mar 2006
Posts: 4,136
P
Megastar
Offline
Megastar
P
Joined: Mar 2006
Posts: 4,136
Quote:
However if the delayed one was the only detector (you covered the normal one) the delayed photon was always seen.


and if there would have been more of the delayed ones
they all would have been seen.

the uncovered one did exactly like the experiment you posted
because all the energy was drained by the collector.

the covered IBM experiment did exactly like the one I
posted except with only 1 medium.

besides if you store the light in glass then wouldnt it be
interesting to see the light dissappear from all the other
19 mediums when all the energy gets drained by a single photo cell.

and if the light is staggered into 20 seperate events then
it would be really hard to understand or explain why the
light that is still visibly trapped INSIDE the other 19 mediums does not have any LIGHT as we know it or understand it that can be detected as it exits the other 19 mediums and impacts the other 19
photo cells.

so either the light is trapped inside the 20 mediums
and registers on all 20 photo cells.

or

the light is trapped inside the 20 mediums and the trapped light energy inside 19 of them is completely drained or
vanish while the light is inside the 19 mediums
and only one completes it journey and registers the full emmission energy.

that would be a great experiment to watch as the light
inside the 19 mediums just vanish because unless the
light that we see is independent from the energy of light
itself then the light that we see traveling inside the
other 19 mediums would have to just vanish as we watch it
on a high speed video playback on youtube.

so there would be a period of time that all of the 20 mediums
have light traveling through them and none of the light has
yet exited any one the 20 mediums , so the light would be
visible and detectable inside all 20 mediums ,

and then when one of the photo cells drains all of the
light energy away there should still be light trapped inside
the other 19 mediums unless the energy of light is independant
from the light of light.

I personally dont think I will ever see light just vanish inside a medium even if the light source is switched off
the light in the medium should continue to travel inside
the medium and exit the medium and speed away from the medium at light speed.

me thinks theres more to it that me is hearing.

its obviously a conspiracy , thats my theory anyway. smile

we see light from stars that may have already burned out billions of years ago , because that light has indeed been trapped inside the medium of space.

now Im trying to think of a really cheap way to check
this conspiracy theory out.




3/4 inch of dust build up on the moon in 4.527 billion years,LOL and QM is fantasy science.
Joined: Dec 2010
Posts: 1,858
B
Megastar
Offline
Megastar
B
Joined: Dec 2010
Posts: 1,858
Originally Posted By: Paul
and if the light is staggered into 20 seperate events then
it would be really hard to understand or explain why the
light that is still visibly trapped INSIDE the other 19 mediums does not have any LIGHT as we know it or understand it that can be detected as it exits the other 19 mediums and impacts the other 19
photo cells.

What would be even harder would be to see the light trapped in the media. The only way to 'see' the light is to detect it. To detect it its energy has to be transferred to the detector. If it is detected in one of the media then there won't be a photon to detect later.

Bill Gill


C is not the speed of light in a vacuum.
C is the universal speed limit.
Joined: Mar 2006
Posts: 4,136
P
Megastar
Offline
Megastar
P
Joined: Mar 2006
Posts: 4,136
Quote:
The only way to 'see' the light is to detect it.


according to orac we cant see a single photon anyway because
it takes a minumum of 8 photons for our eyes to see light.

so a short burst of light for 1 entire second might be
best to use , then measure the emission energy and the
energy collected from the 20 capacitors.

energy out (the source) and energy collected (the photo cells)...

my problem is that all 20 of the photo cells should register
not just one.





3/4 inch of dust build up on the moon in 4.527 billion years,LOL and QM is fantasy science.
Joined: Mar 2006
Posts: 4,136
P
Megastar
Offline
Megastar
P
Joined: Mar 2006
Posts: 4,136
light is generated in packets so light travels in packets.
so obviously there are gaps in light ...

time gaps.

like a light bulb flickers on and off 60 times a second.

or think of a generator in a electrical power plant.

3600 rpm / 60 seconds = 60 hertz ... I think.

1800 rpm / 60 seconds = 30 hertz ...

the light flashes with the frequency of the power source.

so the packets of light from a star may be due to the
frequency of light production on the star.

sounds logical to me anyway.



3/4 inch of dust build up on the moon in 4.527 billion years,LOL and QM is fantasy science.
Joined: Mar 2006
Posts: 4,136
P
Megastar
Offline
Megastar
P
Joined: Mar 2006
Posts: 4,136
Quote:
then there won't be a photon to detect later.


even if you were using a single photon and you did not detect
the light that radiates away from the single photon
as the radiated light enters the 20 mediums and passes
through the 20 mediums with increasing thicknesses and exits
the 20 mediums over 20 different time periods and is absorbed
by the 20 photo cells and is collected and stored in the 20 capacitors, each of the 20 capicitors should register a charge.

so according to what you posted as soon as the light is
detected by the first photo cell the light that is trapped
inside the other 20 glass mediums will simply vanish?

is that your view on this?


3/4 inch of dust build up on the moon in 4.527 billion years,LOL and QM is fantasy science.
Joined: May 2011
Posts: 2,819
O
Megastar
Offline
Megastar
O
Joined: May 2011
Posts: 2,819
Originally Posted By: paul
so according to what you posted as soon as the light is detected by the first photo cell the light that is trapped inside the other 20 glass mediums will simply vanish?

is that your view on this?

It's not my view, IT IS THE EXPERIMENTAL RESULT of every experiment ever done on it. There are literally thousands of them all showing the same thing.

So I find you use of the word "view" rather strange perhaps its a translation thing.

Originally Posted By: paul
its obviously a conspiracy , thats my theory anyway. smile

Not sure what you think the conspiracy is no-one knows how this works we are just reporting the experimental results. I can describe this well under QM but that is about all, so I am unsure why we would conspire to make up the resul. You are basically saying even though there is $1.5 million USD up for grabs to prove it wrong we aren't for a conspiracy reason. I can assure you if I could prove it wrong I would and take the money and run, what do I care what the answer is. So explain to me why I might be conspiring to not report the correct answer please Paul, I am really curious on your logic.

The basic problem is simple to represent the energy of a photon of light has a formula E=HV. H is a constant and V is simply the frequency of the light.

Do you notice what is lacking ... any reference to distance.

E=HV stands whether you are 1cm away of 1 billion light years away.

The macro law says the light power diminishes by a square law function because the surface area of a sphere increases via 4 * pi * r^2.

The macro law and the micro law are in conflict one includes distance and one doesn't.

E=HV still stands for radio waves it just makes the value a lot smaller as the frequency is lower, and we don't think of them as photons. This was my reason for testing them when I was young but much to my disappointment the same results happen. Yes I was going for the Nobel prize I was sure there must be a boundary on the behaviour but I was to be very disappointed.

I am struggling to see how there can be a conspiracy here, solve the problem and fame and money is all yours.

Last edited by Orac; 03/14/16 05:14 AM.

I believe in "Evil, Bad, Ungodly fantasy science and maths", so I am undoubtedly wrong to you.
Joined: May 2011
Posts: 2,819
O
Megastar
Offline
Megastar
O
Joined: May 2011
Posts: 2,819
Originally Posted By: Bill S.

Christopher Baird deals with this, here:

http://sciencequestionswithsurprisingans...en-the-photons/

I would appreciate comments on this.

I think his description is about as good as I have seen for a semi classical view.

Originally Posted By: Christopher Baird
Rather, they are spread out in time and space as they are received.

Not sure any layman is going to understand that, its a fairly complex idea.

Originally Posted By: Christopher Baird
Rather, it receives the light as a series of discrete bundles of energy separated by gaps in time. Similarly, shine light at a sufficiently sensitive array of photon counters, and it receives the light at point locations with spatial gaps between them.

The first part will make sense to a layman but the second part won't .. how does something spreading out appear at a point.

I think you have to describe it as a collapse anything else just seems confusing to a layman.

I agree totally with what he is describing but I am just not sure layman will understand it. Somehow I think you have to deal with this thing spreading out in a sphere can and does collapse to a point.

The other main point Christoper doesn't deal with, is the one Paul is struggling with that even if you take the waveform into different media simultaneously ... the detection of the photon in any of the media collapses all the energy to that point in the one media. It appears like the waveform was never in the other media .... AKA Paul's conspiracy. See paul had no problem with the gap behaviour what he can't get over is this behaviour because he finds it strange and we must be making a mistake or on a conspiracy. You see Bill G understands it as a collapse and correctly identified the experimental result that if you detect it in one media it disappears from everywhere else.

To me it's easier for a layman to equate it with lightning where you have and can measure a sheet voltage but the moment a lightning bolt breaks all the power collapses to a point transmission and the sheet voltage disappears. Maybe I am wrong but I just feel you have to deal with the collapse. Lubos for example would take me to task for making the collapse physical.

The only other way to explain it a layman might understand is using electronic currency where you can use your credit card at any given place in world that supports the credit card , provided you have funds. The problem here is it doesn't deal with the concept of the spherical spread in being able to withdraw the funds.

Last edited by Orac; 03/14/16 05:17 AM.

I believe in "Evil, Bad, Ungodly fantasy science and maths", so I am undoubtedly wrong to you.
Joined: Aug 2010
Posts: 3,570
B
Bill S. Offline OP
Megastar
OP Offline
Megastar
B
Joined: Aug 2010
Posts: 3,570
Originally Posted By: C Baird
…shine light at a sufficiently sensitive array of photon counters, and it receives the light at point locations with spatial gaps between them. When viewed in this way, a light beam always has gaps between its photons, whether the light be very bright or very dim.


Doesn’t this suggest that the spherical spread would collapse into multiple points, which would be detected at a range of places? Presumably the energy of these points (photons?) would equal the energy of the waves.

Would this be an example of the photon appearing as a wave, or particle, depending on the measurement taken?


There never was nothing.
Joined: May 2011
Posts: 2,819
O
Megastar
Offline
Megastar
O
Joined: May 2011
Posts: 2,819
Originally Posted By: Bill S.
Doesn’t this suggest that the spherical spread would collapse into multiple points, which would be detected at a range of places? Presumably the energy of these points (photons?) would equal the energy of the waves.

Would this be an example of the photon appearing as a wave, or particle, depending on the measurement taken?

Yes he doesn't deal with that at all and he sort of avoids discussing the collapse to right at end. Why that doesn't happen we know the answer, it is conservation of energy.

In the above example we have a single photon, if all 20 detectors saw it they are extracting 20 photons of energy. Take the circle bigger and put in 1 billion detectors and 1 photon produces 1 billion photons of energy. Free energy for all, and I could power a whole city off 1 photon, even though I never had that much energy at the source originally smile

What causes the collapse in that context is obvious, the energy can only be consumed at one place. Once consumed the wave in all the other places disappears. That much of the understanding has always been understood but understanding of the process beyond that is limited.

What we did in the above is meticulously dissect the process, and looked at tests we could do. Where we ended was being unable to break the micro experiments and find we had weird answers. It lead us to realize our macro laws must be wrong and can be only approximations based on having an abundance of photons compared to observers. What science is currently doing is trying to confirm that by dismantling all the macro laws.

Last edited by Orac; 03/15/16 02:27 AM.

I believe in "Evil, Bad, Ungodly fantasy science and maths", so I am undoubtedly wrong to you.
Joined: Mar 2006
Posts: 4,136
P
Megastar
Offline
Megastar
P
Joined: Mar 2006
Posts: 4,136
a single photon flash , we cant see it.

several time activated high speed cameras , not flash activated.

what happens?


3/4 inch of dust build up on the moon in 4.527 billion years,LOL and QM is fantasy science.
Joined: Dec 2010
Posts: 1,858
B
Megastar
Offline
Megastar
B
Joined: Dec 2010
Posts: 1,858
Originally Posted By: paul
a single photon flash , we cant see it.

several time activated high speed cameras , not flash activated.

what happens?





One camera will detect the photon. The others will see nothing. Which camera detects the photon is completely random. That of course assumes precisely the same distance from the source to the all of the cameras.

That of course is one of the essences of QM. QM can predict that one of a number of events will happen, and give the probabilities of which one will happen. It cannot predict which one.
Bill Gill


C is not the speed of light in a vacuum.
C is the universal speed limit.
Joined: May 2011
Posts: 2,819
O
Megastar
Offline
Megastar
O
Joined: May 2011
Posts: 2,819
Cheating Bill G, yes the answer is obvious and trivial in QM and matches experimental results but they don't accept or understand QM smile

It can be described by QM but the reason is purely conservation of energy, the initial EM emission only has a discrete value of energy available. The problem is simple (it is the same as my RF harvester example) you reach a point very easily where you have more observers absorbing the energy than you have available at the emission point. With a single photon anything more than one observer creates the condition. If each observer really could get the same energy you would violate conservation of energy and make no mistake that is what is behind the strange behaviour.

Paul doesn't believe in QM and that is fine we don't need to convert him. I encourage and support him trying to find an alternative answer and anything short of denying the experimental results is fine with me. The experiments are dead simple and are repeatable in millions of ways and all give the same result, pretty much because noone has ever been able to beat conservation of energy laws.

The problem even comes up in cameras and eyes which should not work if light is a wave like Paul wanted. With a camera or eye looking at the night sky the wave should come in thru the lense be focused as a spread area on the view area and the whole area should see the light. You wouldn't be able to localize it to a point. Go to an optical site and they have to draw the light as a ray (a transverse wave) to make sense of the results, you cant draw it as a spherical emission wave. The real answer is the spherical wave does enter as Paul imagined, if the whole plate saw the photon and all absorbed it we again have the violation of conservation of energy. The shortest path on probability will see the photon (which matches the ray description) and that pixel triggers and so the result localizes to one point. We in fact have experiments where we fire single photons at a photographic plate and the result is indeed a scattering of the dots the highest probability at the shortest path. So a photographic image is never perfectly clean because it does involve an element of probability. Much harder to explain all that to a layman and so we cheat and turn light to a ray representation when discussing optics. Most layman never pick up there is hand waving involved in normal school level optics.

Prof Christopher Baird who Bill S linked goes the same path I would it is better not to lie and hand wave when discussing this stuff. He also like me stopped short of dragging QM into it because it isn't required for an understanding.

So none of that behaviour requires a QM description and Paul can have his spherical waves so long as he remembers the waves are subject to conservation of energy. Once the energy that was emitted has been absorbed however he wants to describe it the spherical wave must cease everywhere else because there is no energy left. If he does that he will derive the same answer as the experimental results.

For the record the cameras do not have to be the same distance that just changes the probabilities. The closer camera is more likely the further camera less likely but the probability change is steep. That result is also consistent with a single photon fired at a photographic plate.

Last edited by Orac; 03/31/16 05:16 AM.

I believe in "Evil, Bad, Ungodly fantasy science and maths", so I am undoubtedly wrong to you.
Joined: Mar 2006
Posts: 4,136
P
Megastar
Offline
Megastar
P
Joined: Mar 2006
Posts: 4,136
Quote:
One camera will detect the photon. The others will see nothing.


I don't know about you guys but to me that just does not
sound logical at all.

I can understand why a single photo cell would consume
the light energy because the photo cell converts the light energy into electricity.

but a cameras film does not convert light energy into
electricity.

so the light energy is not consumed by the film.

if anything the light is being absorbed and re-emitted
by the film unless camera film actually does consume light energy.

isnt it a chemical reaction to light that causes the film
in a camera to form images on the film?

if the camera film does not consume light energy and
the light energy is being absorbed and re-emmitted then all
of the cameras should capture the photon flash.

because the energy of the photon flash would not be
being drained and should not collapse the field as in the
photo cell experiments.


3/4 inch of dust build up on the moon in 4.527 billion years,LOL and QM is fantasy science.
Joined: Mar 2006
Posts: 4,136
P
Megastar
Offline
Megastar
P
Joined: Mar 2006
Posts: 4,136
suppose a camera were constructed that would take a
360 degree panoramic picture of the flash , the film itself
stretched 360 degrees around the flash so that as the light
sphere reached the film the film would capture the image from
all directions that the light flash traveled in.

would there be a constant line on the film of the
captured image or a single dot?

you wouldn't really need a camera at all just a roll of
unexposed film stretched around the flash point , it might need to be unrolled in a circle around the flash point while
inside a dark room to avoid pre-exposure to any light source
before the single photon light flash is executed.







3/4 inch of dust build up on the moon in 4.527 billion years,LOL and QM is fantasy science.
Joined: May 2011
Posts: 2,819
O
Megastar
Offline
Megastar
O
Joined: May 2011
Posts: 2,819
Originally Posted By: paul
but a cameras film does not convert light energy into electricity.

A camera film works on silver halide it takes energy to change the structure which you develop.

https://en.wikipedia.org/wiki/Photographic_film

Quote:
a very short exposure to the image formed by a camera lens is used to produce only a very slight chemical change, proportional to the amount of light absorbed by each crystal.

So you next statement is wrong
Originally Posted By: paul
so the light energy is not consumed by the film. <=== ERROR

Then a bunch of errors follow on which I will leave you to walk thru.

The film works EXACTLY the same way as your eye it absorbs the energy of your wave, end of story for the energy as it was needed to change the crystal. That is how photography works and why xrays for example create an image because some of the xrays get absorbed by the silver halide and leave a latent image to develop.

Last edited by Orac; 03/31/16 04:41 PM.

I believe in "Evil, Bad, Ungodly fantasy science and maths", so I am undoubtedly wrong to you.
Joined: May 2011
Posts: 2,819
O
Megastar
Offline
Megastar
O
Joined: May 2011
Posts: 2,819
Originally Posted By: paul
suppose a camera were constructed that would take a
360 degree panoramic picture of the flash , the film itself
stretched 360 degrees around the flash so that as the light
sphere reached the film the film would capture the image from
all directions that the light flash traveled in.

would there be a constant line on the film of the
captured image or a single dot?

you wouldn't really need a camera at all just a roll of
unexposed film stretched around the flash point , it might need to be unrolled in a circle around the flash point while
inside a dark room to avoid pre-exposure to any light source
before the single photon light flash is executed.

Now you are really thinking like a scientist and the answer for a single photon is a single dot.

Do you get that if that wasn't so your eye wouldn't work because you would see each photon across the entire eye? Look again at your answer above, your eye is only probably 120-180 degree but it meets that exact criteria you just described. The back of eye is basically a 3D hemisphere version of a rolled out film and exactly what you described.

So how can you see an individual point of light on your eye. The lense could bring the wave to a one of single point (like using a magnifying glass to burn things) but it cant create dots across an area ... see the problem? So put two points of light how does you eye see two individual points in your idea above, they should flood the entire eye and join together just like you say should happen to the film.

So if I accept your answer then I can't see smile

As I said there is nothing wrong with your spherical wave idea you just have to accept it collapses and you will get the right result which matches experiment. At school they just hack the result and talk about wave/particle duality to avoid discussing this collapse situation.

The answer isn't obvious in classical physics but you can see there aren't really any other choices. Like it or not any simple experiment throws up this problem and the naive answer is wrong because of conservation of energy.

This is what makes teaching light to layman hard, you see Marosz fall down this hole continually. All this stuff is dead easy to test and has been done since 17th century. The problem was formally solved in 1855 by James Maxwell although the collapse part had some time before its significance became obvious.

Last edited by Orac; 03/31/16 04:40 PM.

I believe in "Evil, Bad, Ungodly fantasy science and maths", so I am undoubtedly wrong to you.
Page 1 of 9 1 2 3 4 5 6 7 8 9

Link Copied to Clipboard
Newest Members
debbieevans, bkhj, jackk, Johnmattison, RacerGT
865 Registered Users
Sponsor

Science a GoGo's Home Page | Terms of Use | Privacy Policy | Contact UsokÂþ»­¾W
Features | News | Books | Physics | Space | Climate Change | Health | Technology | Natural World

Copyright © 1998 - 2016 Science a GoGo and its licensors. All rights reserved.

Powered by UBB.threads™ PHP Forum Software 7.7.5